Search Results

Search found 6242 results on 250 pages for 'named pipe'.

Page 38/250 | < Previous Page | 34 35 36 37 38 39 40 41 42 43 44 45  | Next Page >

  • Locating Rogue Perl Script

    - by Gary Garside
    I've been trying to source the location of a perl script which is causing havoc on a server which i control. I'm also trying to find out exactly how this script was installed on the server - my best guess is through a wordpress exploit. The server is a basic web setup running Ubuntu 9.04, Apache and MySQL. I use IPTables for firewall, the site runs around 20 sites and the load never really creeps above 0.7. From what i can see the script is making outbound connection to other servers (most likely trying to brute force entry). Here is a top dump of one of the processes: PID USER PR NI VIRT RES SHR S %CPU %MEM TIME+ COMMAND 22569 www-data 20 0 22784 3216 780 R 100 0.2 47:00.60 perl The command the process is running is /usr/sbin/sshd . I've tried to find an exact file name but im having no luck... i've ran a lsof -p PID and here is the output: COMMAND PID USER FD TYPE DEVICE SIZE NODE NAME perl 22569 www-data cwd DIR 8,6 4096 2 / perl 22569 www-data rtd DIR 8,6 4096 2 / perl 22569 www-data txt REG 8,6 10336 162220 /usr/bin/perl perl 22569 www-data mem REG 8,6 26936 170219 /usr/lib/perl/5.10.0/auto/Socket/Socket.so perl 22569 www-data mem REG 8,6 22808 170214 /usr/lib/perl/5.10.0/auto/IO/IO.so perl 22569 www-data mem REG 8,6 39112 145112 /lib/libcrypt-2.9.so perl 22569 www-data mem REG 8,6 1502512 145124 /lib/libc-2.9.so perl 22569 www-data mem REG 8,6 130151 145113 /lib/libpthread-2.9.so perl 22569 www-data mem REG 8,6 542928 145122 /lib/libm-2.9.so perl 22569 www-data mem REG 8,6 14608 145125 /lib/libdl-2.9.so perl 22569 www-data mem REG 8,6 1503704 162222 /usr/lib/libperl.so.5.10.0 perl 22569 www-data mem REG 8,6 135680 145116 /lib/ld-2.9.so perl 22569 www-data 0r FIFO 0,6 157216 pipe perl 22569 www-data 1w FIFO 0,6 197642 pipe perl 22569 www-data 2w FIFO 0,6 197642 pipe perl 22569 www-data 3w FIFO 0,6 197642 pipe perl 22569 www-data 4u IPv4 383991 TCP outsidesoftware.com:56869->server12.34.56.78.live-servers.net:www (ESTABLISHED) My gut feeling is outsidesoftware.com is also under attacK? Or possibly being used as a tunnel. I've managed to find a number of rouge files in /tmp and /var/tmp, here is a brief output of one of these files: #!/usr/bin/perl # this spreader is coded by xdh # xdh@xxxxxxxxxxx # only for testing... my @nickname = ("vn"); my $nick = $nickname[rand scalar @nickname]; my $ircname = $nickname[rand scalar @nickname]; #system("kill -9 `ps ax |grep httpdse |grep -v grep|awk '{print $1;}'`"); my $processo = '/usr/sbin/sshd'; The full file contents can be viewed here: http://pastebin.com/yenFRrGP Im trying to achieve a couple of things here... Firstly i need to stop these processes from running. Either by disabling outbound SSH or any IP Tables rules etc... these scripts have been running for around 36 hours now and my main concern is to stop these things running and respawning by themselves. Secondly i need to try and source where and how these scripts have been installed. If anybody has any advise on what to look for in access logs or anything else i would be really grateful. Thanks in advance

    Read the article

  • Courier Maildrop error user unknown. Command output: Invalid user specified

    - by cad
    Hello I have a problem with maildrop. I have read dozens of webs/howto/emails but couldnt solve it. My objective is moving automatically spam messages to a spam folder. My email server is working perfectly. It marks spam in subject and headers using spamassasin. My box has: Ubuntu 9.04 Web: Apache2 + Php5 + MySQL MTA: Postfix 2.5.5 + SpamAssasin + virtual users using mysql IMAP: Courier 0.61.2 + Courier AuthLib WebMail: SquirrelMail I have read that I could use Squirrelmail directly (not a good idea), procmail or maildrop. As I already have maildrop in the box (from courier) I have configured the server to use maildrop (added an entry in transport table for a virtual domain). I found this error in email: This is the mail system at host foo.net I'm sorry to have to inform you that your message could not be delivered to one or more recipients. It's attached below. For further assistance, please send mail to postmaster. If you do so, please include this problem report. You can delete your own text from the attached returned message. The mail system <[email protected]>: user unknown. Command output: Invalid user specified. Final-Recipient: rfc822; [email protected] Action: failed Status: 5.1.1 Diagnostic-Code: x-unix; Invalid user specified. ---------- Forwarded message ---------- From: test <[email protected]> To: [email protected] Date: Sat, 1 May 2010 19:49:57 +0100 Subject: fail fail An this in the logs May 1 18:50:18 foo.net postfix/smtpd[14638]: connect from mail-bw0-f212.google.com[209.85.218.212] May 1 18:50:19 foo.net postfix/smtpd[14638]: 8A9E9DC23F: client=mail-bw0-f212.google.com[209.85.218.212] May 1 18:50:19 foo.net postfix/cleanup[14643]: 8A9E9DC23F: message-id=<[email protected]> May 1 18:50:19 foo.net postfix/qmgr[14628]: 8A9E9DC23F: from=<[email protected]>, size=1858, nrcpt=1 (queue active) May 1 18:50:23 foo.net postfix/pickup[14627]: 1D4B4DC2AA: uid=5002 from=<[email protected]> May 1 18:50:23 foo.net postfix/cleanup[14643]: 1D4B4DC2AA: message-id=<[email protected]> May 1 18:50:23 foo.net postfix/pipe[14644]: 8A9E9DC23F: to=<[email protected]>, relay=spamassassin, delay=3.8, delays=0.55/0.02/0/3.2, dsn=2.0.0, status=sent (delivered via spamassassin service) May 1 18:50:23 foo.net postfix/qmgr[14628]: 8A9E9DC23F: removed May 1 18:50:23 foo.net postfix/qmgr[14628]: 1D4B4DC2AA: from=<[email protected]>, size=2173, nrcpt=1 (queue active) **May 1 18:50:23 foo.netpostfix/pipe[14648]: 1D4B4DC2AA: to=<[email protected]>, relay=maildrop, delay=0.22, delays=0.06/0.01/0/0.15, dsn=5.1.1, status=bounced (user unknown. Command output: Invalid user specified. )** May 1 18:50:23 foo.net postfix/cleanup[14643]: 4C2BFDC240: message-id=<[email protected]> May 1 18:50:23 foo.net postfix/qmgr[14628]: 4C2BFDC240: from=<>, size=3822, nrcpt=1 (queue active) May 1 18:50:23 foo.net postfix/bounce[14651]: 1D4B4DC2AA: sender non-delivery notification: 4C2BFDC240 May 1 18:50:23 foo.net postfix/qmgr[14628]: 1D4B4DC2AA: removed May 1 18:50:24 foo.net postfix/smtp[14653]: 4C2BFDC240: to=<[email protected]>, relay=gmail-smtp-in.l.google.com[209.85.211.97]:25, delay=0.91, delays=0.02/0.03/0.12/0.74, dsn=2.0.0, status=sent (250 2.0.0 OK 1272739824 37si5422420ywh.59) May 1 18:50:24 foo.net postfix/qmgr[14628]: 4C2BFDC240: removed My config files: http://lar3d.net/main.cf (/etc/postfix) http://lar3d.net/master.c (/etc/postfix) http://lar3d.net/local.cf (/etc/spamassasin) http://lar3d.net/maildroprc (maildroprc) If I change master.cf line (as suggested here) maildrop unix - n n - - pipe flags=DRhu user=vmail argv=/usr/lib/courier/bin/maildrop -d ${recipient} with maildrop unix - n n - - pipe flags=DRhu user=vmail argv=/usr/lib/courier/bin/maildrop -d vmail ${recipient} I get the email in /home/vmail/MailDir instead of the correct dir (/home/vmail/foo.net/info/.SPAM ) After reading a lot I have some guess but not sure. - Maybe I have to install userdb? - Maybe is something related with mysql, but everything is working ok - If I try with procmail I will face same problem... - What are flags DRhu for? Couldnt find doc about them - In some places I found maildrop line with more parameters flags=DRhu user=vmail argv=/usr/lib/courier/bin/maildrop -d $ ${recipient} ${extension} ${recipient} ${user} ${nexthop} ${sender} I am really lost. Dont know how to continue. If you have any idea or need another config file please let me know. Thanks!!!

    Read the article

  • Getting output parameter(SYS_REFCURSOR) from Oracle stored procedure in iBATIS 3(by using annotation

    - by yjacket
    I got an example how to call oracle SP in iBATIS 3 without a map file. And now I understand how to call it. But I got another problem that how to get a result from output parameter(Oracle cursor). A part of exception messages is "There is no setter for property named 'rs' in 'class java.lang.Class". Below is my code. Does anyone can help me? Oracle Stored Procedure: CREATE OR REPLACE PROCEDURE getProducts ( rs OUT SYS_REFCURSOR ) IS BEGIN OPEN rs FOR SELECT * FROM Products; END getProducts; Interface: public interface ProductMapper { @Select("call getProducts(#{rs,mode=OUT,jdbcType=CURSOR})") @Options(statementType = StatementType.CALLABLE) List<Product> getProducts(); } DAO: public class ProductDAO { public List<Product> getProducts() { return mapper.getProducts(); // mapper is ProductMapper } } Full Error Message: Exception in thread "main" org.apache.ibatis.exceptions.IbatisException: ### Error querying database. Cause: org.apache.ibatis.reflection.ReflectionException: Could not set property 'rs' of 'class org.apache.ibatis.reflection.MetaObject$NullObject' with value 'oracle.jdbc.driver.OracleResultSetImpl@1a001ff' Cause: org.apache.ibatis.reflection.ReflectionException: There is no setter for property named 'rs' in 'class java.lang.Class' ### The error may involve defaultParameterMap ### The error occurred while setting parameters ### Cause: org.apache.ibatis.reflection.ReflectionException: Could not set property 'rs' of 'class org.apache.ibatis.reflection.MetaObject$NullObject' with value 'oracle.jdbc.driver.OracleResultSetImpl@1a001ff' Cause: org.apache.ibatis.reflection.ReflectionException: There is no setter for property named 'rs' in 'class java.lang.Class' at org.apache.ibatis.exceptions.ExceptionFactory.wrapException(ExceptionFactory.java:8) at org.apache.ibatis.session.defaults.DefaultSqlSession.selectList(DefaultSqlSession.java:61) at org.apache.ibatis.session.defaults.DefaultSqlSession.selectList(DefaultSqlSession.java:53) at org.apache.ibatis.binding.MapperMethod.executeForList(MapperMethod.java:82) at org.apache.ibatis.binding.MapperMethod.execute(MapperMethod.java:63) at org.apache.ibatis.binding.MapperProxy.invoke(MapperProxy.java:35) at $Proxy8.getList(Unknown Source) at com.dao.ProductDAO.getList(ProductDAO.java:42) at com.Ibatis3Test.main(Ibatis3Test.java:30) Caused by: org.apache.ibatis.reflection.ReflectionException: Could not set property 'rs' of 'class org.apache.ibatis.reflection.MetaObject$NullObject' with value 'oracle.jdbc.driver.OracleResultSetImpl@1a001ff' Cause: org.apache.ibatis.reflection.ReflectionException: There is no setter for property named 'rs' in 'class java.lang.Class' at org.apache.ibatis.reflection.wrapper.BeanWrapper.setBeanProperty(BeanWrapper.java:154) at org.apache.ibatis.reflection.wrapper.BeanWrapper.set(BeanWrapper.java:36) at org.apache.ibatis.reflection.MetaObject.setValue(MetaObject.java:120) at org.apache.ibatis.executor.resultset.FastResultSetHandler.handleOutputParameters(FastResultSetHandler.java:69) at org.apache.ibatis.executor.statement.CallableStatementHandler.query(CallableStatementHandler.java:44) at org.apache.ibatis.executor.statement.RoutingStatementHandler.query(RoutingStatementHandler.java:55) at org.apache.ibatis.executor.SimpleExecutor.doQuery(SimpleExecutor.java:41) at org.apache.ibatis.executor.BaseExecutor.query(BaseExecutor.java:94) at org.apache.ibatis.executor.CachingExecutor.query(CachingExecutor.java:72) at org.apache.ibatis.session.defaults.DefaultSqlSession.selectList(DefaultSqlSession.java:59) ... 7 more Caused by: org.apache.ibatis.reflection.ReflectionException: There is no setter for property named 'rs' in 'class java.lang.Class' at org.apache.ibatis.reflection.Reflector.getSetInvoker(Reflector.java:300) at org.apache.ibatis.reflection.MetaClass.getSetInvoker(MetaClass.java:97) at org.apache.ibatis.reflection.wrapper.BeanWrapper.setBeanProperty(BeanWrapper.java:146) ... 16 more

    Read the article

  • How to get a result from output parameter(SYS_REFCURSOR) of Oracle stored procedure in iBATIS 3(by u

    - by yjacket
    I got an example how to call oracle SP in iBATIS 3 without a map file. And now I understand how to call it. But I got another problem that how to get a result from output parameter(Oracle cursor). A part of exception messages is "There is no setter for property named 'rs' in 'class java.lang.Class". Below is my code. Does anyone can help me? Oracle Stored Procedure: CREATE OR REPLACE PROCEDURE getProducts ( rs OUT SYS_REFCURSOR ) IS BEGIN OPEN rs FOR SELECT * FROM Products; END getProducts; Interface: public interface ProductMapper { @Select("call getProducts(#{rs,mode=OUT,jdbcType=CURSOR})") @Options(statementType = StatementType.CALLABLE) List<Product> getProducts(); } DAO: public class ProductDAO { public List<Product> getProducts() { return mapper.getProducts(); // mapper is ProductMapper } } Full Error Message: Exception in thread "main" org.apache.ibatis.exceptions.IbatisException: ### Error querying database. Cause: org.apache.ibatis.reflection.ReflectionException: Could not set property 'rs' of 'class org.apache.ibatis.reflection.MetaObject$NullObject' with value 'oracle.jdbc.driver.OracleResultSetImpl@1a001ff' Cause: org.apache.ibatis.reflection.ReflectionException: There is no setter for property named 'rs' in 'class java.lang.Class' ### The error may involve defaultParameterMap ### The error occurred while setting parameters ### Cause: org.apache.ibatis.reflection.ReflectionException: Could not set property 'rs' of 'class org.apache.ibatis.reflection.MetaObject$NullObject' with value 'oracle.jdbc.driver.OracleResultSetImpl@1a001ff' Cause: org.apache.ibatis.reflection.ReflectionException: There is no setter for property named 'rs' in 'class java.lang.Class' at org.apache.ibatis.exceptions.ExceptionFactory.wrapException(ExceptionFactory.java:8) at org.apache.ibatis.session.defaults.DefaultSqlSession.selectList(DefaultSqlSession.java:61) at org.apache.ibatis.session.defaults.DefaultSqlSession.selectList(DefaultSqlSession.java:53) at org.apache.ibatis.binding.MapperMethod.executeForList(MapperMethod.java:82) at org.apache.ibatis.binding.MapperMethod.execute(MapperMethod.java:63) at org.apache.ibatis.binding.MapperProxy.invoke(MapperProxy.java:35) at $Proxy8.getList(Unknown Source) at com.dao.ProductDAO.getList(ProductDAO.java:42) at com.Ibatis3Test.main(Ibatis3Test.java:30) Caused by: org.apache.ibatis.reflection.ReflectionException: Could not set property 'rs' of 'class org.apache.ibatis.reflection.MetaObject$NullObject' with value 'oracle.jdbc.driver.OracleResultSetImpl@1a001ff' Cause: org.apache.ibatis.reflection.ReflectionException: There is no setter for property named 'rs' in 'class java.lang.Class' at org.apache.ibatis.reflection.wrapper.BeanWrapper.setBeanProperty(BeanWrapper.java:154) at org.apache.ibatis.reflection.wrapper.BeanWrapper.set(BeanWrapper.java:36) at org.apache.ibatis.reflection.MetaObject.setValue(MetaObject.java:120) at org.apache.ibatis.executor.resultset.FastResultSetHandler.handleOutputParameters(FastResultSetHandler.java:69) at org.apache.ibatis.executor.statement.CallableStatementHandler.query(CallableStatementHandler.java:44) at org.apache.ibatis.executor.statement.RoutingStatementHandler.query(RoutingStatementHandler.java:55) at org.apache.ibatis.executor.SimpleExecutor.doQuery(SimpleExecutor.java:41) at org.apache.ibatis.executor.BaseExecutor.query(BaseExecutor.java:94) at org.apache.ibatis.executor.CachingExecutor.query(CachingExecutor.java:72) at org.apache.ibatis.session.defaults.DefaultSqlSession.selectList(DefaultSqlSession.java:59) ... 7 more Caused by: org.apache.ibatis.reflection.ReflectionException: There is no setter for property named 'rs' in 'class java.lang.Class' at org.apache.ibatis.reflection.Reflector.getSetInvoker(Reflector.java:300) at org.apache.ibatis.reflection.MetaClass.getSetInvoker(MetaClass.java:97) at org.apache.ibatis.reflection.wrapper.BeanWrapper.setBeanProperty(BeanWrapper.java:146) ... 16 more

    Read the article

  • Coexistence of projects between Visual Studio 2010 and 2012

    - by sreejukg
    Microsoft has released another version of Visual Studio named Visual Studio 2012. As you can see there are user interface (UI) changes in all/most of the Microsoft applications as Microsoft is moving towards Windows 8 and changing the UI scheme for all of the applications. Visual Studio 2012 is a move to adapt the new interface requirements that are in coherent with Windows 8. Not only this Visual Studio 2012 has lots of improvements in several areas and it supports .Net framework 4.5. In the past, whenever a new version of Visual Studio launches, developers needed to upgrade the project to new version of Visual Studio which was a pain, especially when you are working with a team of developers. Once a solution is upgraded to a newer version, it was not possible to going back. With Visual studio 2012, you can avoid the pain of upgrading. Developers will be able to open their project in Visual Studio 2012 along with Visual Studio 2010 SP 1. This means if you create a project using Visual Studio 2012, you will be able to open it with Visual Studio 2010 SP 1 and vice versa. There are some exceptions (as always!). Visual Studio 2012 supports some new project types, which was not there in 2010 version. Such project, you will not be able to open in Visual Studio 2010. For e.g. Visual Studio 2012 brings a new project type named “Windows 8 Modern Applications”, such projects you will not be able to open using the 2010 version of Visual Studio. Just to prove the said subject, I am going to perform some simple operations. I installed Visual Studio 2010 with SP 1 and Visual Studio 2012 on my PC. See the snapshots for both the installations. Visual Studio 2010 Visual Studio 2012 Now I am going to perform two test cases. First create a project in 2010 Version and open it in 2012 version and vice versa. If you are interested, you can continue scrolling down, otherwise just say bye bye to this article. Case 1: Open a solution created using Visual Studio 2010 in 2012 version. I created a project in VS 2010 named TestProject2010 using empty ASP.Net web application template. Once created the project appears in VS 2010 as follows. I closed Visual Studio and opened the solution file using VS 2012 by using the Open Project dialog(File -> Open Project/Solution). Surprisingly, there is not even a warning message, just the project opened fine in Visual Studio 2012. Case 2: Open a solution created using Visual Studio 2012 in 2010 version. I have created a project in Visual Studio 2012 named testProject2012. See the screenshot of the project in VS 2012 below. Now try opening the solution in Visual Studio 2010. The solution loaded successfully, but Visual Studio failed to load project. See the screenshot. At first I was surprised. The Web application project template is available in both versions, So there should not be any problem. What is making the incompatibility? Is it ASP.Net version? Yes it is. VS 2012 assign ASP.Net 4.5 as the default version that was causing the trouble for Visual Studio 2010. I changed the version to .Net framework 4.0 and saved the project after that I was able to open the project in Visual Studio 2010. This as an excellent move from Visual Studio Team and allows enterprises to perform gradual upgrade to the new version. Now developers can work in any version based on availability and preference, simply I can use Visual Studio 2012 as my IDE while my colleague working on the same project can still use Visual Studio 2010.

    Read the article

  • Ubuntu 12.04 patched b43 driver compilation error

    - by Zed
    I tried this How do I install this patched b43 driver? guide to install patched b43 driver on Ubuntu 12.04 with 3.2.0-31-generic kernel but I can't pass compilation phase.Here is what I did: wget http://www.orbit-lab.org/kernel/compat-wireless-3-stable/v3.1/compat-wireless-3.1.1-1.tar.bz2 cd compat-wireless-3.1.1-1/ scripts/driver-select b43 make make -C /lib/modules/3.2.0-31-generic/build M=/home/marco/compat-wireless-3.1.1-1 modules make[1]: Entering directory `/usr/src/linux-headers-3.2.0-31-generic' CC [M] /home/marco/compat-wireless-3.1.1-1/compat/main.o In file included from /home/marco/compat-wireless-3.1.1-1/include/linux/compat-2.6.29.h:5:0, from /home/marco/compat-wireless-3.1.1-1/include/linux/compat-2.6.h:24, from <command-line>:0: include/linux/netdevice.h:1153:5: warning: "IS_ENABLED" is not defined [-Wundef] include/linux/netdevice.h:1153:15: error: missing binary operator before token "(" include/linux/netdevice.h: In function ‘netdev_uses_dsa_tags’: include/linux/netdevice.h:1421:9: error: ‘struct net_device’ has no member named ‘dsa_ptr’ include/linux/netdevice.h:1422:31: error: ‘struct net_device’ has no member named ‘dsa_ptr’ include/linux/netdevice.h: In function ‘netdev_uses_trailer_tags’: include/linux/netdevice.h:1431:9: error: ‘struct net_device’ has no member named ‘dsa_ptr’ include/linux/netdevice.h:1432:35: error: ‘struct net_device’ has no member named ‘dsa_ptr’ make[3]: *** [/home/marco/compat-wireless-3.1.1-1/compat/main.o] Error 1 make[2]: *** [/home/marco/compat-wireless-3.1.1-1/compat] Error 2 make[1]: *** [_module_/home/marco/compat-wireless-3.1.1-1] Error 2 make[1]: Leaving directory `/usr/src/linux-headers-3.2.0-31-generic' make: *** [modules] Error 2 To fix that error I added #include <linux/kconfig.h> to /usr/src/linux-headers-3.2.0-31-generic/include/linux/netdevice.h but now I'm getting something else make make -C /lib/modules/3.2.0-31-generic/build M=/home/marco/compat-wireless-3.1.1-1 modules make[1]: Entering directory `/usr/src/linux-headers-3.2.0-31-generic' CC [M] /home/marco/compat-wireless-3.1.1-1/compat/main.o LD [M] /home/marco/compat-wireless-3.1.1-1/compat/compat.o CC [M] /home/marco/compat-wireless-3.1.1-1/drivers/bcma/main.o In file included from /home/marco/compat-wireless-3.1.1-1/include/linux/bcma/bcma.h:9:0, from /home/marco/compat-wireless-3.1.1-1/drivers/bcma/bcma_private.h:8, from /home/marco/compat-wireless-3.1.1-1/drivers/bcma/main.c:8: /home/marco/compat-wireless-3.1.1-1/include/linux/ssb/ssb.h: In function ‘ssb_driver_register’: /home/marco/compat-wireless-3.1.1-1/include/linux/ssb/ssb.h:236:36: error: ‘THIS_MODULE’ undeclared (first use in this function) /home/marco/compat-wireless-3.1.1-1/include/linux/ssb/ssb.h:236:36: note: each undeclared identifier is reported only once for each function it appears in In file included from /home/marco/compat-wireless-3.1.1-1/drivers/bcma/bcma_private.h:8:0, from /home/marco/compat-wireless-3.1.1-1/drivers/bcma/main.c:8: /home/marco/compat-wireless-3.1.1-1/include/linux/bcma/bcma.h: In function ‘bcma_driver_register’: /home/marco/compat-wireless-3.1.1-1/include/linux/bcma/bcma.h:170:37: error: ‘THIS_MODULE’ undeclared (first use in this function) /home/marco/compat-wireless-3.1.1-1/drivers/bcma/main.c: At top level: /home/marco/compat-wireless-3.1.1-1/drivers/bcma/main.c:12:20: error: expected declaration specifiers or ‘...’ before string constant /home/marco/compat-wireless-3.1.1-1/drivers/bcma/main.c:13:16: error: expected declaration specifiers or ‘...’ before string constant /home/marco/compat-wireless-3.1.1-1/drivers/bcma/main.c:182:1: warning: data definition has no type or storage class [enabled by default] /home/marco/compat-wireless-3.1.1-1/drivers/bcma/main.c:182:1: warning: type defaults to ‘int’ in declaration of ‘EXPORT_SYMBOL_GPL’ [-Wimplicit-int] /home/marco/compat-wireless-3.1.1-1/drivers/bcma/main.c:182:1: warning: parameter names (without types) in function declaration [enabled by default] /home/marco/compat-wireless-3.1.1-1/drivers/bcma/main.c:188:1: warning: data definition has no type or storage class [enabled by default] /home/marco/compat-wireless-3.1.1-1/drivers/bcma/main.c:188:1: warning: type defaults to ‘int’ in declaration of ‘EXPORT_SYMBOL_GPL’ [-Wimplicit-int] /home/marco/compat-wireless-3.1.1-1/drivers/bcma/main.c:188:1: warning: parameter names (without types) in function declaration [enabled by default] make[3]: *** [/home/marco/compat-wireless-3.1.1-1/drivers/bcma/main.o] Error 1 make[2]: *** [/home/marco/compat-wireless-3.1.1-1/drivers/bcma] Error 2 make[1]: *** [_module_/home/marco/compat-wireless-3.1.1-1] Error 2 make[1]: Leaving directory `/usr/src/linux-headers-3.2.0-31-generic' make: *** [modules] Error 2 Any suggestion what to try next?

    Read the article

  • Ubuntu server random hangups.

    - by Ebbe
    Hello all. this is my first post to this forum which I found through the superb podcast "It Conversations" from StackOverFlow. I am quite in my role as server administrator for an exhibition center in London. Basically we have a central file and sql server to which roughly 40 stations connects to to upload/download data used/captured by a set of applications. Over the last weeks we have experienced a few random hangups to our applications, and as it always happen to multiple applications simultaneously I do not believe that the applications are the source of the problem. We also monitor the network using Dartware Intermapper which indicates that all switches and stations on the network has been reachable during the downtime. Thus, its all pointing to the server. I have been looking through all log files I can think of and the only thing so far that I have found suspicious is the following lines in the syslog which are from the time of one of the hangups: Feb 6 17:14:27 es named[5582]: client 127.0.0.1#33721: RFC 1918 response from Internet for 150.0.168.192.in-addr.arpa Feb 6 17:14:40 es named[5582]: client 127.0.0.1#32899: RFC 1918 response from Internet for 152.0.168.192.in-addr.arpa Feb 6 17:15:01 es /USR/SBIN/CRON[1956]: (es) CMD (/home/es/apps/es/bin/es_checksum.sh) Feb 6 17:16:06 es /USR/SBIN/CRON[2031]: (es) CMD (/home/es/apps/es/bin/es_checksum.sh) Feb 6 17:21:00 es named[5582]: *** POKED TIMER *** Feb 6 17:21:00 es last message repeated 2 times Feb 6 17:21:07 es named[5582]: client 127.0.0.1#44194: RFC 1918 response from Internet for 143.0.168.192.in-addr.arpa Feb 6 17:21:12 es named[5582]: client 127.0.0.1#59004: RFC 1918 response from Internet for 164.0.168.192.in-addr.arpa I find a few lines of interesting lines here: 1) "RFC 1918 response from Internet for 150.1.168.192.in-addr.arpa". I see this a lot in the syslog. And basically everytime I do a nslookup for any of the computers in the cluster I get a new similar line in the syslog. I understand from google that this has to do with reverse lookup problems. But I do not know how that could effect the systems. Lets say that one of these lines appear every time one of the userstations connects to the server, which may happen several times a second. Could this possible cause a hangup of the entire server? 2) POKED TIMER, I have googled this quite a lot, but not found an explaination that I can relate to. What does this mean? 3) The timestamps, it seems like the entire server has stopped responding for several minutes. Normally there are many printouts to the syslog per minute on this server. Furthermore the CRON job is set to run once every minute. Which according to the log, hasent happened here. OS: Ubuntu 8.04 Kernel: Linux 2.6.24-24-server x86_64 GNU/Linux. Hardware: Dell R710, RAID1, CPU: 2x XEON E5530. 16GB Memory. Average load is very low, and memory should not be a problem. Please let me know if you need any additional information. Best wishes, Ebbe

    Read the article

  • C# 5 Async, Part 2: Asynchrony Today

    - by Reed
    The .NET Framework has always supported asynchronous operations.  However, different mechanisms for supporting exist throughout the framework.  While there are at least three separate asynchronous patterns used through the framework, only the latest is directly usable with the new Visual Studio Async CTP.  Before delving into details on the new features, I will talk about existing asynchronous code, and demonstrate how to adapt it for use with the new pattern. The first asynchronous pattern used in the .NET framework was the Asynchronous Programming Model (APM).  This pattern was based around callbacks.  A method is used to start the operation.  It typically is named as BeginSomeOperation.  This method is passed a callback defined as an AsyncCallback, and returns an object that implements IAsyncResult.  Later, the IAsyncResult is used in a call to a method named EndSomeOperation, which blocks until completion and returns the value normally directly returned from the synchronous version of the operation.  Often, the EndSomeOperation call would be called from the callback function passed, which allows you to write code that never blocks. While this pattern works perfectly to prevent blocking, it can make quite confusing code, and be difficult to implement.  For example, the sample code provided for FileStream’s BeginRead/EndRead methods is not simple to understand.  In addition, implementing your own asynchronous methods requires creating an entire class just to implement the IAsyncResult. Given the complexity of the APM, other options have been introduced in later versions of the framework.  The next major pattern introduced was the Event-based Asynchronous Pattern (EAP).  This provides a simpler pattern for asynchronous operations.  It works by providing a method typically named SomeOperationAsync, which signals its completion via an event typically named SomeOperationCompleted. The EAP provides a simpler model for asynchronous programming.  It is much easier to understand and use, and far simpler to implement.  Instead of requiring a custom class and callbacks, the standard event mechanism in C# is used directly.  For example, the WebClient class uses this extensively.  A method is used, such as DownloadDataAsync, and the results are returned via the DownloadDataCompleted event. While the EAP is far simpler to understand and use than the APM, it is still not ideal.  By separating your code into method calls and event handlers, the logic of your program gets more complex.  It also typically loses the ability to block until the result is received, which is often useful.  Blocking often requires writing the code to block by hand, which is error prone and adds complexity. As a result, .NET 4 introduced a third major pattern for asynchronous programming.  The Task<T> class introduced a new, simpler concept for asynchrony.  Task and Task<T> effectively represent an operation that will complete at some point in the future.  This is a perfect model for thinking about asynchronous code, and is the preferred model for all new code going forward.  Task and Task<T> provide all of the advantages of both the APM and the EAP models – you have the ability to block on results (via Task.Wait() or Task<T>.Result), and you can stay completely asynchronous via the use of Task Continuations.  In addition, the Task class provides a new model for task composition and error and cancelation handling.  This is a far superior option to the previous asynchronous patterns. The Visual Studio Async CTP extends the Task based asynchronous model, allowing it to be used in a much simpler manner.  However, it requires the use of Task and Task<T> for all operations.

    Read the article

  • Linux servers going into Halt when pressing Control-D in putty or exit in the shell

    - by Itai Ganot
    Since today at noon, there's a number of Linux CentOS servers which are going to Halt whenever i type exit or use Control-D to close the putty window. Did anyone encounter this weird behavior before? I've checked the aliases list on the servers and there is no alias regarding halt command. After the server came online i've checked the history and saw a "logout" command there but nothing which is related to Halt. At first, i thought it happens only from my computer but later i realized that it happens to everyone which types exit, logout or control+d. 2 of these server are our main iptables firewalls and so it's super critical, your assistance is much appreciated. It looks like that, and it only happens on servers with active IPTables: [root@srv1 bin]# ssh srv2 root@srv2's password: Last login: Sun Nov 11 17:19:41 2012 from 192.168.12.98 [root@srv2 ~]# vim /etc/crontab [root@srv2 ~]# exit logout Broadcast message from root (pts/1) (Tue Nov 13 10:44:04 2012): The system is going down for system halt NOW! Connection to srv2 closed. [root@srv1 bin]# In my troubleshooting steps i came across the command strace, and so i've opened two bash windows to one of the problematic server and i used strace -p PID_of_bash. When i typed in exit in the first shell it did go to halt, attached is the strace output, if you can check it out and tell me if you see anything suspicious i'd be more than thankful. RER, 0x2b0e45a8f2f0}, 8) = 0 rt_sigaction(SIGALRM, {0x4484f0, [HUP INT ILL TRAP ABRT BUS FPE USR1 SEGV USR2 PIPE ALRM TERM XCPU XFSZ VTALRM SYS], SA_RESTORER, 0x2b0e45a8f2f0}, {0x47c450, [], SA_RESTORER, 0x2b0e45a8f2f0}, 8) = 0 rt_sigaction(SIGTSTP, {0x1, [], SA_RESTORER, 0x2b0e45a8f2f0}, {0x1, [], SA_RESTORER, 0x2b0e45a8f2f0}, 8) = 0 rt_sigaction(SIGTTOU, {0x1, [], SA_RESTORER, 0x2b0e45a8f2f0}, {0x1, [], SA_RESTORER, 0x2b0e45a8f2f0}, 8) = 0 rt_sigaction(SIGTTIN, {0x1, [], SA_RESTORER, 0x2b0e45a8f2f0}, {0x1, [], SA_RESTORER, 0x2b0e45a8f2f0}, 8) = 0 rt_sigaction(SIGWINCH, {0x448370, [], SA_RESTORER, 0x2b0e45a8f2f0}, {0x47c410, [], SA_RESTORER|SA_RESTART, 0x2b0e45a8f2f0}, 8) = 0 socket(PF_NETLINK, SOCK_RAW, 9) = 3 sendmsg(3, {msg_name(12)={sa_family=AF_NETLINK, pid=0, groups=00000000}, msg_iov(2)=[{"\25\0\0\0d\4\1\0\0\0\0\0\0\0\0\0", 16}, {"exit\0", 5}], msg_controllen=0, msg_flags=0}, 0) = 21 close(3) = 0 rt_sigaction(SIGINT, {0x448700, [], SA_RESTORER, 0x2b0e45a8f2f0}, {0x448700, [], SA_RESTORER, 0x2b0e45a8f2f0}, 8) = 0 write(2, "logout\n", 7) = 7 write(2, "There are stopped jobs.\n", 24) = 24 rt_sigprocmask(SIG_BLOCK, NULL, [], 8) = 0 rt_sigaction(SIGINT, {0x448700, [], SA_RESTORER, 0x2b0e45a8f2f0}, {0x448700, [], SA_RESTORER, 0x2b0e45a8f2f0}, 8) = 0 rt_sigprocmask(SIG_BLOCK, NULL, [], 8) = 0 rt_sigprocmask(SIG_BLOCK, [INT CHLD], [], 8) = 0 pipe([3, 4]) = 0 clone(child_stack=0, flags=CLONE_CHILD_CLEARTID|CLONE_CHILD_SETTID|SIGCHLD, child_tidptr=0x2b0e45db6fe0) = 23717 setpgid(23717, 23717) = 0 rt_sigprocmask(SIG_SETMASK, [], NULL, 8) = 0 rt_sigprocmask(SIG_BLOCK, [CHLD], [], 8) = 0 close(3) = 0 close(4) = 0 rt_sigprocmask(SIG_BLOCK, [CHLD TSTP TTIN TTOU], [CHLD], 8) = 0 ioctl(255, TIOCSPGRP, [23717]) = 0 rt_sigprocmask(SIG_SETMASK, [CHLD], NULL, 8) = 0 rt_sigprocmask(SIG_SETMASK, [], NULL, 8) = 0 rt_sigprocmask(SIG_BLOCK, [CHLD], [], 8) = 0 wait4(-1, [{WIFEXITED(s) && WEXITSTATUS(s) == 0}], WSTOPPED|WCONTINUED, NULL) = 23717 rt_sigprocmask(SIG_BLOCK, [CHLD TSTP TTIN TTOU], [CHLD], 8) = 0 ioctl(255, TIOCSPGRP, [20458]) = 0 rt_sigprocmask(SIG_SETMASK, [CHLD], NULL, 8) = 0 ioctl(255, SNDCTL_TMR_TIMEBASE or TCGETS, {B38400 opost isig icanon echo ...}) = 0 ioctl(255, TIOCGWINSZ, {ws_row=53, ws_col=211, ws_xpixel=0, ws_ypixel=0}) = 0 rt_sigprocmask(SIG_SETMASK, [], NULL, 8) = 0 --- SIGCHLD (Child exited) @ 0 (0) --- wait4(-1, 0x7fff395da984, WNOHANG|WSTOPPED|WCONTINUED, NULL) = 0 rt_sigreturn(0x11) = 0 rt_sigprocmask(SIG_BLOCK, [CHLD TSTP TTIN TTOU], [], 8) = 0 ioctl(255, TIOCSPGRP, [20458]) = 0 rt_sigprocmask(SIG_SETMASK, [], NULL, 8) = 0 rt_sigaction(SIGINT, {0x448700, [], SA_RESTORER, 0x2b0e45a8f2f0}, {0x448700, [], SA_RESTORER, 0x2b0e45a8f2f0}, 8) = 0 rt_sigprocmask(SIG_BLOCK, [INT], [], 8) = 0 ioctl(0, TIOCGWINSZ, {ws_row=53, ws_col=211, ws_xpixel=0, ws_ypixel=0}) = 0 ioctl(0, TIOCSWINSZ, {ws_row=53, ws_col=211, ws_xpixel=0, ws_ypixel=0}) = 0 ioctl(0, SNDCTL_TMR_TIMEBASE or TCGETS, {B38400 opost isig icanon echo ...}) = 0 ioctl(0, SNDCTL_TMR_STOP or TCSETSW, {B38400 opost isig -icanon -echo ...}) = 0 rt_sigprocmask(SIG_SETMASK, [], NULL, 8) = 0 rt_sigprocmask(SIG_BLOCK, [INT QUIT ALRM TSTP TTIN TTOU], [], 8) = 0 rt_sigaction(SIGINT, {0x47c450, [], SA_RESTORER, 0x2b0e45a8f2f0}, {0x448700, [], SA_RESTORER, 0x2b0e45a8f2f0}, 8) = 0 rt_sigaction(SIGTERM, {0x47c450, [], SA_RESTORER, 0x2b0e45a8f2f0}, {0x1, [], SA_RESTORER, 0x2b0e45a8f2f0}, 8) = 0 rt_sigaction(SIGTERM, {0x1, [], SA_RESTORER, 0x2b0e45a8f2f0}, {0x47c450, [], SA_RESTORER, 0x2b0e45a8f2f0}, 8) = 0 rt_sigaction(SIGQUIT, {0x47c450, [], SA_RESTORER, 0x2b0e45a8f2f0}, {0x1, [], SA_RESTORER, 0x2b0e45a8f2f0}, 8) = 0 rt_sigaction(SIGQUIT, {0x1, [], SA_RESTORER, 0x2b0e45a8f2f0}, {0x47c450, [], SA_RESTORER, 0x2b0e45a8f2f0}, 8) = 0 rt_sigaction(SIGALRM, {0x47c450, [], SA_RESTORER, 0x2b0e45a8f2f0}, {0x4484f0, [HUP INT ILL TRAP ABRT BUS FPE USR1 SEGV USR2 PIPE ALRM TERM XCPU XFSZ VTALRM SYS], SA_RESTORER, 0x2b0e45a8f2f0}, 8) = 0 rt_sigaction(SIGTSTP, {0x47c450, [], SA_RESTORER, 0x2b0e45a8f2f0}, {0x1, [], SA_RESTORER, 0x2b0e45a8f2f0}, 8) = 0 rt_sigaction(SIGTSTP, {0x1, [], SA_RESTORER, 0x2b0e45a8f2f0}, {0x47c450, [], SA_RESTORER, 0x2b0e45a8f2f0}, 8) = 0 rt_sigaction(SIGTTOU, {0x47c450, [], SA_RESTORER, 0x2b0e45a8f2f0}, {0x1, [], SA_RESTORER, 0x2b0e45a8f2f0}, 8) = 0 rt_sigaction(SIGTTOU, {0x1, [], SA_RESTORER, 0x2b0e45a8f2f0}, {0x47c450, [], SA_RESTORER, 0x2b0e45a8f2f0}, 8) = 0 rt_sigaction(SIGTTIN, {0x47c450, [], SA_RESTORER, 0x2b0e45a8f2f0}, {0x1, [], SA_RESTORER, 0x2b0e45a8f2f0}, 8) = 0 rt_sigaction(SIGTTIN, {0x1, [], SA_RESTORER, 0x2b0e45a8f2f0}, {0x47c450, [], SA_RESTORER, 0x2b0e45a8f2f0}, 8) = 0 rt_sigprocmask(SIG_SETMASK, [], NULL, 8) = 0 rt_sigaction(SIGWINCH, {0x47c410, [], SA_RESTORER|SA_RESTART, 0x2b0e45a8f2f0}, {0x448370, [], SA_RESTORER, 0x2b0e45a8f2f0}, 8) = 0 write(2, "[root@g2-lga ~]# ", 17) = 17 rt_sigprocmask(SIG_BLOCK, NULL, [], 8) = 0 read(0, "e", 1) = 1 write(2, "e", 1) = 1 rt_sigprocmask(SIG_BLOCK, NULL, [], 8) = 0 read(0, "x", 1) = 1 write(2, "x", 1) = 1 rt_sigprocmask(SIG_BLOCK, NULL, [], 8) = 0 read(0, "i", 1) = 1 write(2, "i", 1) = 1 rt_sigprocmask(SIG_BLOCK, NULL, [], 8) = 0 read(0, "t", 1) = 1 write(2, "t", 1) = 1 rt_sigprocmask(SIG_BLOCK, NULL, [], 8) = 0 read(0, "\r", 1) = 1 write(2, "\n", 1) = 1 rt_sigprocmask(SIG_BLOCK, [INT], [], 8) = 0 ioctl(0, SNDCTL_TMR_STOP or TCSETSW, {B38400 opost isig icanon echo ...}) = 0 rt_sigprocmask(SIG_SETMASK, [], NULL, 8) = 0 rt_sigaction(SIGINT, {0x448700, [], SA_RESTORER, 0x2b0e45a8f2f0}, {0x47c450, [], SA_RESTORER, 0x2b0e45a8f2f0}, 8) = 0 rt_sigaction(SIGTERM, {0x1, [], SA_RESTORER, 0x2b0e45a8f2f0}, {0x1, [], SA_RESTORER, 0x2b0e45a8f2f0}, 8) = 0 rt_sigaction(SIGQUIT, {0x1, [], SA_RESTORER, 0x2b0e45a8f2f0}, {0x1, [], SA_RESTORER, 0x2b0e45a8f2f0}, 8) = 0 rt_sigaction(SIGALRM, {0x4484f0, [HUP INT ILL TRAP ABRT BUS FPE USR1 SEGV USR2 PIPE ALRM TERM XCPU XFSZ VTALRM SYS], SA_RESTORER, 0x2b0e45a8f2f0}, {0x47c450, [], SA_RESTORER, 0x2b0e45a8f2f0}, 8) = 0 rt_sigaction(SIGTSTP, {0x1, [], SA_RESTORER, 0x2b0e45a8f2f0}, {0x1, [], SA_RESTORER, 0x2b0e45a8f2f0}, 8) = 0 rt_sigaction(SIGTTOU, {0x1, [], SA_RESTORER, 0x2b0e45a8f2f0}, {0x1, [], SA_RESTORER, 0x2b0e45a8f2f0}, 8) = 0 rt_sigaction(SIGTTIN, {0x1, [], SA_RESTORER, 0x2b0e45a8f2f0}, {0x1, [], SA_RESTORER, 0x2b0e45a8f2f0}, 8) = 0 rt_sigaction(SIGWINCH, {0x448370, [], SA_RESTORER, 0x2b0e45a8f2f0}, {0x47c410, [], SA_RESTORER|SA_RESTART, 0x2b0e45a8f2f0}, 8) = 0 socket(PF_NETLINK, SOCK_RAW, 9) = 3 sendmsg(3, {msg_name(12)={sa_family=AF_NETLINK, pid=0, groups=00000000}, msg_iov(2)=[{"\25\0\0\0d\4\1\0\0\0\0\0\0\0\0\0", 16}, {"exit\0", 5}], msg_controllen=0, msg_flags=0}, 0) = 21 close(3) = 0 rt_sigaction(SIGINT, {0x448700, [], SA_RESTORER, 0x2b0e45a8f2f0}, {0x448700, [], SA_RESTORER, 0x2b0e45a8f2f0}, 8) = 0 write(2, "logout\n", 7) = 7 open("/root/.bash_logout", O_RDONLY) = 3 fstat(3, {st_mode=S_IFREG|0644, st_size=24, ...}) = 0 read(3, "# ~/.bash_logout\n\nclear\n", 24) = 24 close(3) = 0 rt_sigprocmask(SIG_BLOCK, NULL, [], 8) = 0 rt_sigprocmask(SIG_BLOCK, NULL, [], 8) = 0 rt_sigprocmask(SIG_BLOCK, NULL, [], 8) = 0 rt_sigprocmask(SIG_BLOCK, NULL, [], 8) = 0 stat(".", {st_mode=S_IFDIR|0750, st_size=12288, ...}) = 0 stat("/usr/kerberos/sbin/clear", 0x7fff395da960) = -1 ENOENT (No such file or directory) stat("/usr/kerberos/bin/clear", 0x7fff395da960) = -1 ENOENT (No such file or directory) stat("/usr/local/sbin/clear", 0x7fff395da960) = -1 ENOENT (No such file or directory) stat("/usr/local/bin/clear", 0x7fff395da960) = -1 ENOENT (No such file or directory) stat("/sbin/clear", 0x7fff395da960) = -1 ENOENT (No such file or directory) stat("/bin/clear", 0x7fff395da960) = -1 ENOENT (No such file or directory) stat("/usr/sbin/clear", 0x7fff395da960) = -1 ENOENT (No such file or directory) stat("/usr/bin/clear", {st_mode=S_IFREG|0755, st_size=12712, ...}) = 0 access("/usr/bin/clear", X_OK) = 0 access("/usr/bin/clear", R_OK) = 0 stat("/usr/bin/clear", {st_mode=S_IFREG|0755, st_size=12712, ...}) = 0 access("/usr/bin/clear", X_OK) = 0 access("/usr/bin/clear", R_OK) = 0 rt_sigprocmask(SIG_BLOCK, [INT CHLD], [], 8) = 0 pipe([3, 4]) = 0 clone(child_stack=0, flags=CLONE_CHILD_CLEARTID|CLONE_CHILD_SETTID|SIGCHLD, child_tidptr=0x2b0e45db6fe0) = 23726 setpgid(23726, 23726) = 0 rt_sigprocmask(SIG_SETMASK, [], NULL, 8) = 0 rt_sigprocmask(SIG_BLOCK, [CHLD], [], 8) = 0 close(3) = 0 close(4) = 0 rt_sigprocmask(SIG_BLOCK, [CHLD TSTP TTIN TTOU], [CHLD], 8) = 0 ioctl(255, TIOCSPGRP, [23726]) = 0 rt_sigprocmask(SIG_SETMASK, [CHLD], NULL, 8) = 0 rt_sigprocmask(SIG_SETMASK, [], NULL, 8) = 0 rt_sigprocmask(SIG_BLOCK, [CHLD], [], 8) = 0 wait4(-1, Broadcast message from root (pts/0) (Wed Nov 14 12:41:44 2012): The system is going down for system halt NOW! [{WIFEXITED(s) && WEXITSTATUS(s) == 0}], WSTOPPED|WCONTINUED, NULL) = 23726 rt_sigprocmask(SIG_BLOCK, [CHLD TSTP TTIN TTOU], [CHLD], 8) = 0 ioctl(255, TIOCSPGRP, [20458]) = 0 rt_sigprocmask(SIG_SETMASK, [CHLD], NULL, 8) = 0 ioctl(255, SNDCTL_TMR_TIMEBASE or TCGETS, {B38400 opost isig icanon echo ...}) = 0 ioctl(255, TIOCGWINSZ, {ws_row=53, ws_col=211, ws_xpixel=0, ws_ypixel=0}) = 0 rt_sigprocmask(SIG_SETMASK, [], NULL, 8) = 0 --- SIGCHLD (Child exited) @ 0 (0) --- wait4(-1, 0x7fff395da634, WNOHANG|WSTOPPED|WCONTINUED, NULL) = 0 rt_sigreturn(0x11) = 0 open("/etc/bash.bash_logout", O_RDONLY) = -1 ENOENT (No such file or directory) rt_sigprocmask(SIG_SETMASK, [], NULL, 8) = 0 rt_sigaction(SIGINT, {0x448700, [], SA_RESTORER, 0x2b0e45a8f2f0}, {0x448700, [], SA_RESTORER, 0x2b0e45a8f2f0}, 8) = 0 stat("/root/.bash_history", {st_mode=S_IFREG|0600, st_size=28900, ...}) = 0 open("/root/.bash_history", O_WRONLY|O_APPEND) = 3 write(3, "cd /etc/profile.d/\nls\nls -alrt\ng"..., 1120) = 1120 close(3) = 0 open("/root/.bash_history", O_RDONLY) = 3 fstat(3, {st_mode=S_IFREG|0600, st_size=30020, ...}) = 0 read(3, "history \nping g1-lga\nping g1-lga"..., 30020) = 30020 close(3) = 0 open("/root/.bash_history", O_WRONLY|O_TRUNC) = 3 write(3, "grep \"216.18\" *\nhistory \nexit\nvi"..., 27609) = 27609 close(3) = 0 kill(4294965658, SIGTERM) = 0 kill(4294965658, SIGCONT) = 0 --- SIGCHLD (Child exited) @ 0 (0) --- wait4(-1, [{WIFSIGNALED(s) && WTERMSIG(s) == SIGTERM}], WNOHANG|WSTOPPED|WCONTINUED, NULL) = 1638 wait4(-1, 0x7fff395dac34, WNOHANG|WSTOPPED|WCONTINUED, NULL) = -1 ECHILD (No child processes) rt_sigreturn(0x11) = 0 --- SIGCHLD (Child exited) @ 0 (0) --- wait4(-1, 0x7fff395dac34, WNOHANG|WSTOPPED|WCONTINUED, NULL) = -1 ECHILD (No child processes) rt_sigreturn(0xffffffffffffffff) = 0 rt_sigprocmask(SIG_BLOCK, [CHLD TSTP TTIN TTOU], [], 8) = 0 ioctl(255, TIOCSPGRP, [20458]) = 0 rt_sigprocmask(SIG_SETMASK, [], NULL, 8) = 0 setpgid(0, 20458) = -1 EPERM (Operation not permitted) exit_group(1) = ? Process 20458 detached [root@g2-lga ~]#

    Read the article

  • Wicket, Spring and Hibernate - Testing with Unitils - Error: Table not found in statement [select re

    - by John
    Hi there. I've been following a tutorial and a sample application, namely 5 Days of Wicket - Writing the tests: http://www.mysticcoders.com/blog/2009/03/10/5-days-of-wicket-writing-the-tests/ I've set up my own little project with a simple shoutbox that saves messages to a database. I then wanted to set up a couple of tests that would make sure that if a message is stored in the database, the retrieved object would contain the exact same data. Upon running mvn test all my tests fail. The exception has been pasted in the first code box underneath. I've noticed that even though my unitils.properties says to use the 'hdqldb'-dialect, this message is still output in the console window when starting the tests: INFO - Dialect - Using dialect: org.hibernate.dialect.PostgreSQLDialect. I've added the entire dump from the console as well at the bottom of this post (which goes on for miles and miles :-)). Upon running mvn test all my tests fail, and the exception is: Caused by: java.sql.SQLException: Table not found in statement [select relname from pg_class] at org.hsqldb.jdbc.Util.sqlException(Unknown Source) at org.hsqldb.jdbc.jdbcStatement.fetchResult(Unknown Source) at org.hsqldb.jdbc.jdbcStatement.executeQuery(Unknown Source) at org.apache.commons.dbcp.DelegatingStatement.executeQuery(DelegatingStatement.java:188) at org.hibernate.tool.hbm2ddl.DatabaseMetadata.initSequences(DatabaseMetadata.java:151) at org.hibernate.tool.hbm2ddl.DatabaseMetadata.(DatabaseMetadata.java:69) at org.hibernate.tool.hbm2ddl.DatabaseMetadata.(DatabaseMetadata.java:62) at org.springframework.orm.hibernate3.LocalSessionFactoryBean$3.doInHibernate(LocalSessionFactoryBean.java:958) at org.springframework.orm.hibernate3.HibernateTemplate.doExecute(HibernateTemplate.java:419) ... 49 more I've set up my unitils.properties file like so: database.driverClassName=org.hsqldb.jdbcDriver database.url=jdbc:hsqldb:mem:PUBLIC database.userName=sa database.password= database.dialect=hsqldb database.schemaNames=PUBLIC My abstract IntegrationTest class: @SpringApplicationContext({"/com/upbeat/shoutbox/spring/applicationContext.xml", "applicationContext-test.xml"}) public abstract class AbstractIntegrationTest extends UnitilsJUnit4 { private ApplicationContext applicationContext; } applicationContext-test.xml: <?xml version="1.0" encoding="UTF-8"? <beans xmlns="http://www.springframework.org/schema/beans" xmlns:xsi="http://www.w3.org/2001/XMLSchema-instance" xmlns:tx="http://www.springframework.org/schema/tx" xsi:schemaLocation=" http://www.springframework.org/schema/beans http://www.springframework.org/schema/beans/spring-beans-2.5.xsd http://www.springframework.org/schema/tx http://www.springframework.org/schema/tx/spring-tx-2.5.xsd" <bean id="dataSource" class="org.unitils.database.UnitilsDataSourceFactoryBean"/ </beans and finally, one of the test classes: package com.upbeat.shoutbox.web; import org.apache.wicket.spring.injection.annot.test.AnnotApplicationContextMock; import org.apache.wicket.util.tester.WicketTester; import org.junit.Before; import org.junit.Test; import org.unitils.spring.annotation.SpringBeanByType; import com.upbeat.shoutbox.HomePage; import com.upbeat.shoutbox.integrations.AbstractIntegrationTest; import com.upbeat.shoutbox.persistence.ShoutItemDao; import com.upbeat.shoutbox.services.ShoutService; public class TestHomePage extends AbstractIntegrationTest { @SpringBeanByType private ShoutService svc; @SpringBeanByType private ShoutItemDao dao; protected WicketTester tester; @Before public void setUp() { AnnotApplicationContextMock appctx = new AnnotApplicationContextMock(); appctx.putBean("shoutItemDao", dao); appctx.putBean("shoutService", svc); tester = new WicketTester(); } @Test public void testRenderMyPage() { //start and render the test page tester.startPage(HomePage.class); //assert rendered page class tester.assertRenderedPage(HomePage.class); //assert rendered label component tester.assertLabel("message", "If you see this message wicket is properly configured and running"); } } Dump from console when running mvn test: [INFO] Scanning for projects... [INFO] ------------------------------------------------------------------------ [INFO] Building shoutbox [INFO] task-segment: [test] [INFO] ------------------------------------------------------------------------ [INFO] [resources:resources {execution: default-resources}] [WARNING] File encoding has not been set, using platform encoding Cp1252, i.e. build is platform dependent! [WARNING] Using platform encoding (Cp1252 actually) to copy filtered resources, i.e. build is platform dependent! [INFO] Copying 3 resources [INFO] Copying 4 resources [INFO] [compiler:compile {execution: default-compile}] [INFO] Nothing to compile - all classes are up to date [INFO] [resources:testResources {execution: default-testResources}] [WARNING] File encoding has not been set, using platform encoding Cp1252, i.e. build is platform dependent! [WARNING] Using platform encoding (Cp1252 actually) to copy filtered resources, i.e. build is platform dependent! [INFO] Copying 2 resources [INFO] [compiler:testCompile {execution: default-testCompile}] [INFO] Nothing to compile - all classes are up to date [INFO] [surefire:test {execution: default-test}] [INFO] Surefire report directory: F:\Projects\shoutbox\target\surefire-reports INFO - ConfigurationLoader - Loaded main configuration file unitils-default.properties from classpath. INFO - ConfigurationLoader - Loaded custom configuration file unitils.properties from classpath. INFO - ConfigurationLoader - No local configuration file unitils-local.properties found. ------------------------------------------------------- T E S T S ------------------------------------------------------- Running com.upbeat.shoutbox.web.TestViewShoutsPage Tests run: 1, Failures: 0, Errors: 1, Skipped: 0, Time elapsed: 0.02 sec INFO - Version - Hibernate Annotations 3.4.0.GA INFO - Environment - Hibernate 3.3.0.SP1 INFO - Environment - hibernate.properties not found INFO - Environment - Bytecode provider name : javassist INFO - Environment - using JDK 1.4 java.sql.Timestamp handling INFO - Version - Hibernate Commons Annotations 3.1.0.GA INFO - AnnotationBinder - Binding entity from annotated class: com.upbeat.shoutbox.models.ShoutItem INFO - QueryBinder - Binding Named query: item.getById = from ShoutItem item where item.id = :id INFO - QueryBinder - Binding Named query: item.find = from ShoutItem item order by item.timestamp desc INFO - QueryBinder - Binding Named query: item.count = select count(item) from ShoutItem item INFO - EntityBinder - Bind entity com.upbeat.shoutbox.models.ShoutItem on table SHOUT_ITEMS INFO - AnnotationConfiguration - Hibernate Validator not found: ignoring INFO - notationSessionFactoryBean - Building new Hibernate SessionFactory INFO - earchEventListenerRegister - Unable to find org.hibernate.search.event.FullTextIndexEventListener on the classpath. Hibernate Search is not enabled. INFO - ConnectionProviderFactory - Initializing connection provider: org.springframework.orm.hibernate3.LocalDataSourceConnectionProvider INFO - SettingsFactory - RDBMS: HSQL Database Engine, version: 1.8.0 INFO - SettingsFactory - JDBC driver: HSQL Database Engine Driver, version: 1.8.0 INFO - Dialect - Using dialect: org.hibernate.dialect.PostgreSQLDialect INFO - TransactionFactoryFactory - Transaction strategy: org.springframework.orm.hibernate3.SpringTransactionFactory INFO - actionManagerLookupFactory - No TransactionManagerLookup configured (in JTA environment, use of read-write or transactional second-level cache is not recommended) INFO - SettingsFactory - Automatic flush during beforeCompletion(): disabled INFO - SettingsFactory - Automatic session close at end of transaction: disabled INFO - SettingsFactory - JDBC batch size: 1000 INFO - SettingsFactory - JDBC batch updates for versioned data: disabled INFO - SettingsFactory - Scrollable result sets: enabled INFO - SettingsFactory - JDBC3 getGeneratedKeys(): disabled INFO - SettingsFactory - Connection release mode: auto INFO - SettingsFactory - Default batch fetch size: 1 INFO - SettingsFactory - Generate SQL with comments: disabled INFO - SettingsFactory - Order SQL updates by primary key: disabled INFO - SettingsFactory - Order SQL inserts for batching: disabled INFO - SettingsFactory - Query translator: org.hibernate.hql.ast.ASTQueryTranslatorFactory INFO - ASTQueryTranslatorFactory - Using ASTQueryTranslatorFactory INFO - SettingsFactory - Query language substitutions: {} INFO - SettingsFactory - JPA-QL strict compliance: disabled INFO - SettingsFactory - Second-level cache: enabled INFO - SettingsFactory - Query cache: enabled INFO - SettingsFactory - Cache region factory : org.hibernate.cache.impl.bridge.RegionFactoryCacheProviderBridge INFO - FactoryCacheProviderBridge - Cache provider: org.hibernate.cache.HashtableCacheProvider INFO - SettingsFactory - Optimize cache for minimal puts: disabled INFO - SettingsFactory - Structured second-level cache entries: disabled INFO - SettingsFactory - Query cache factory: org.hibernate.cache.StandardQueryCacheFactory INFO - SettingsFactory - Echoing all SQL to stdout INFO - SettingsFactory - Statistics: disabled INFO - SettingsFactory - Deleted entity synthetic identifier rollback: disabled INFO - SettingsFactory - Default entity-mode: pojo INFO - SettingsFactory - Named query checking : enabled INFO - SessionFactoryImpl - building session factory INFO - essionFactoryObjectFactory - Not binding factory to JNDI, no JNDI name configured INFO - UpdateTimestampsCache - starting update timestamps cache at region: org.hibernate.cache.UpdateTimestampsCache INFO - StandardQueryCache - starting query cache at region: org.hibernate.cache.StandardQueryCache INFO - notationSessionFactoryBean - Updating database schema for Hibernate SessionFactory INFO - Dialect - Using dialect: org.hibernate.dialect.PostgreSQLDialect INFO - XmlBeanDefinitionReader - Loading XML bean definitions from class path resource [org/springframework/jdbc/support/sql-error-codes.xml] INFO - SQLErrorCodesFactory - SQLErrorCodes loaded: [DB2, Derby, H2, HSQL, Informix, MS-SQL, MySQL, Oracle, PostgreSQL, Sybase] INFO - DefaultListableBeanFactory - Destroying singletons in org.springframework.beans.factory.support.DefaultListableBeanFactory@3e0ebb: defining beans [propertyConfigurer,dataSource,sessionFactory,shoutService,shoutItemDao,wicketApplication,org.springframework.aop.config.internalAutoProxyCreator,org.springframework.transaction.annotation.AnnotationTransactionAttributeSource#0,org.springframework.transaction.interceptor.TransactionInterceptor#0,org.springframework.transaction.config.internalTransactionAdvisor,transactionManager]; root of factory hierarchy INFO - sPathXmlApplicationContext - Refreshing org.springframework.context.support.ClassPathXmlApplicationContext@a8e586: display name [org.springframework.context.support.ClassPathXmlApplicationContext@a8e586]; startup date [Tue May 04 18:19:58 CEST 2010]; root of context hierarchy INFO - XmlBeanDefinitionReader - Loading XML bean definitions from class path resource [com/upbeat/shoutbox/spring/applicationContext.xml] INFO - XmlBeanDefinitionReader - Loading XML bean definitions from class path resource [applicationContext-test.xml] INFO - DefaultListableBeanFactory - Overriding bean definition for bean 'dataSource': replacing [Generic bean: class [org.apache.commons.dbcp.BasicDataSource]; scope=singleton; abstract=false; lazyInit=false; autowireMode=0; dependencyCheck=0; autowireCandidate=true; primary=false; factoryBeanName=null; factoryMethodName=null; initMethodName=null; destroyMethodName=close; defined in class path resource [com/upbeat/shoutbox/spring/applicationContext.xml]] with [Generic bean: class [org.unitils.database.UnitilsDataSourceFactoryBean]; scope=singleton; abstract=false; lazyInit=false; autowireMode=0; dependencyCheck=0; autowireCandidate=true; primary=false; factoryBeanName=null; factoryMethodName=null; initMethodName=null; destroyMethodName=null; defined in class path resource [applicationContext-test.xml]] INFO - sPathXmlApplicationContext - Bean factory for application context [org.springframework.context.support.ClassPathXmlApplicationContext@a8e586]: org.springframework.beans.factory.support.DefaultListableBeanFactory@5dfaf1 INFO - pertyPlaceholderConfigurer - Loading properties file from class path resource [application.properties] INFO - DefaultListableBeanFactory - Pre-instantiating singletons in org.springframework.beans.factory.support.DefaultListableBeanFactory@5dfaf1: defining beans [propertyConfigurer,dataSource,sessionFactory,shoutService,shoutItemDao,wicketApplication,org.springframework.aop.config.internalAutoProxyCreator,org.springframework.transaction.annotation.AnnotationTransactionAttributeSource#0,org.springframework.transaction.interceptor.TransactionInterceptor#0,org.springframework.transaction.config.internalTransactionAdvisor,transactionManager]; root of factory hierarchy INFO - AnnotationBinder - Binding entity from annotated class: com.upbeat.shoutbox.models.ShoutItem INFO - QueryBinder - Binding Named query: item.getById = from ShoutItem item where item.id = :id INFO - QueryBinder - Binding Named query: item.find = from ShoutItem item order by item.timestamp desc INFO - QueryBinder - Binding Named query: item.count = select count(item) from ShoutItem item INFO - EntityBinder - Bind entity com.upbeat.shoutbox.models.ShoutItem on table SHOUT_ITEMS INFO - AnnotationConfiguration - Hibernate Validator not found: ignoring INFO - notationSessionFactoryBean - Building new Hibernate SessionFactory INFO - earchEventListenerRegister - Unable to find org.hibernate.search.event.FullTextIndexEventListener on the classpath. Hibernate Search is not enabled. INFO - ConnectionProviderFactory - Initializing connection provider: org.springframework.orm.hibernate3.LocalDataSourceConnectionProvider INFO - SettingsFactory - RDBMS: HSQL Database Engine, version: 1.8.0 INFO - SettingsFactory - JDBC driver: HSQL Database Engine Driver, version: 1.8.0 INFO - Dialect - Using dialect: org.hibernate.dialect.PostgreSQLDialect INFO - TransactionFactoryFactory - Transaction strategy: org.springframework.orm.hibernate3.SpringTransactionFactory INFO - actionManagerLookupFactory - No TransactionManagerLookup configured (in JTA environment, use of read-write or transactional second-level cache is not recommended) INFO - SettingsFactory - Automatic flush during beforeCompletion(): disabled INFO - SettingsFactory - Automatic session close at end of transaction: disabled INFO - SettingsFactory - JDBC batch size: 1000 INFO - SettingsFactory - JDBC batch updates for versioned data: disabled INFO - SettingsFactory - Scrollable result sets: enabled INFO - SettingsFactory - JDBC3 getGeneratedKeys(): disabled INFO - SettingsFactory - Connection release mode: auto INFO - SettingsFactory - Default batch fetch size: 1 INFO - SettingsFactory - Generate SQL with comments: disabled INFO - SettingsFactory - Order SQL updates by primary key: disabled INFO - SettingsFactory - Order SQL inserts for batching: disabled INFO - SettingsFactory - Query translator: org.hibernate.hql.ast.ASTQueryTranslatorFactory INFO - ASTQueryTranslatorFactory - Using ASTQueryTranslatorFactory INFO - SettingsFactory - Query language substitutions: {} INFO - SettingsFactory - JPA-QL strict compliance: disabled INFO - SettingsFactory - Second-level cache: enabled INFO - SettingsFactory - Query cache: enabled INFO - SettingsFactory - Cache region factory : org.hibernate.cache.impl.bridge.RegionFactoryCacheProviderBridge INFO - FactoryCacheProviderBridge - Cache provider: org.hibernate.cache.HashtableCacheProvider INFO - SettingsFactory - Optimize cache for minimal puts: disabled INFO - SettingsFactory - Structured second-level cache entries: disabled INFO - SettingsFactory - Query cache factory: org.hibernate.cache.StandardQueryCacheFactory INFO - SettingsFactory - Echoing all SQL to stdout INFO - SettingsFactory - Statistics: disabled INFO - SettingsFactory - Deleted entity synthetic identifier rollback: disabled INFO - SettingsFactory - Default entity-mode: pojo INFO - SettingsFactory - Named query checking : enabled INFO - SessionFactoryImpl - building session factory INFO - essionFactoryObjectFactory - Not binding factory to JNDI, no JNDI name configured INFO - UpdateTimestampsCache - starting update timestamps cache at region: org.hibernate.cache.UpdateTimestampsCache INFO - StandardQueryCache - starting query cache at region: org.hibernate.cache.StandardQueryCache INFO - notationSessionFactoryBean - Updating database schema for Hibernate SessionFactory INFO - Dialect - Using dialect: org.hibernate.dialect.PostgreSQLDialect INFO - DefaultListableBeanFactory - Destroying singletons in org.springframework.beans.factory.support.DefaultListableBeanFactory@5dfaf1: defining beans [propertyConfigurer,dataSource,sessionFactory,shoutService,shoutItemDao,wicketApplication,org.springframework.aop.config.internalAutoProxyCreator,org.springframework.transaction.annotation.AnnotationTransactionAttributeSource#0,org.springframework.transaction.interceptor.TransactionInterceptor#0,org.springframework.transaction.config.internalTransactionAdvisor,transactionManager]; root of factory hierarchy Tests run: 1, Failures: 0, Errors: 1, Skipped: 0, Time elapsed: 1.34 sec <<< FAILURE! Running com.upbeat.shoutbox.integrations.ShoutItemIntegrationTest Tests run: 1, Failures: 0, Errors: 1, Skipped: 0, Time elapsed: 0 sec <<< FAILURE! Running com.upbeat.shoutbox.mocks.ShoutServiceTest Tests run: 1, Failures: 0, Errors: 1, Skipped: 0, Time elapsed: 0.01 sec <<< FAILURE! Results : Tests in error: initializationError(com.upbeat.shoutbox.web.TestViewShoutsPage) testRenderMyPage(com.upbeat.shoutbox.web.TestHomePage) initializationError(com.upbeat.shoutbox.integrations.ShoutItemIntegrationTest) initializationError(com.upbeat.shoutbox.mocks.ShoutServiceTest) Tests run: 4, Failures: 0, Errors: 4, Skipped: 0 [INFO] ------------------------------------------------------------------------ [ERROR] BUILD FAILURE [INFO] ------------------------------------------------------------------------ [INFO] There are test failures. Please refer to F:\Projects\shoutbox\target\surefire-reports for the individual test results. [INFO] ------------------------------------------------------------------------ [INFO] For more information, run Maven with the -e switch [INFO] ------------------------------------------------------------------------ [INFO] Total time: 3 seconds [INFO] Finished at: Tue May 04 18:19:58 CEST 2010 [INFO] Final Memory: 13M/31M [INFO] ------------------------------------------------------------------------ Any help is greatly appreciated.

    Read the article

  • Where does lucene .net cache the search results?

    - by Lanceomagnifico
    Hi, I'm trying to figure out where Lucene stores the cached query results, and how it's configured to do so - and how long it caches for. This is for an ASP.NET 3.5 solution. I'm getting this problem: If I run a search and sort the result by a particular product field, it seems to work the very first time each search and sort combination is used. If I then go in and change some product attributes, reindex and run the same search and sort, I get the products returned in the same order as the very first result. example Product A is named: foo Product B is named: bar For the first search, sort by name desc. This results in: Product A Product B Now mix up the data a bit: Change names to: Product A named: bar Product B named: foo reindex verify that the index contains the changes for these two products. search Result: Product A Product B Since I changed the alphabetical order of the names, I expected: Product B Product A So I think that Lucene is caching the search results. (Which, btw, is a very good thing.) I just need to know where/how to clear these results. I've tried deleting the index files and doing an IISreset to clear the memory, but it seems to have no effect. So I'm thinking there is another set of Lucene files outside of the indexes that Lucene uses for caching. EDIT I just found out that you must create the index for field you wish to sort on as un-tokenized. I had the field as tokenized, so sorting didn't work.

    Read the article

  • Excel vba -get ActiveX Control checkbox when event handler is triggered

    - by danoran
    I have an excel spreadsheet that is separated into different sections with named ranges. I want to hide a named range when a checkbox is clicked. I can do this for one checkbox, but I would like to have a single function that can hide the appropriate section based on the calling checkbox. I was planning on calling that function from the event_handlers for when the checkboxes are clicked, and to pass the checkbox as an argument. Is there a way to access the checkbox object that calls the event handler? This works: Sub chkDogsInContest_Click() ActiveSheet.Names("DogsInContest").RefersToRange.EntireRow.Hidden = Not chkMemberData.Value End Sub But this is what I would like to do: Sub chkDogsInContest_Click() Module1.Show_Hide_Section (<calling checkbox>) End Sub These functions are defined in a different module: 'The format for the the names of the checkbox controls is 'CHECKBOX_NAME_PREFIX + <name> 'where "name" is also the name of the associated Named Range Public Const CHECKBOX_NAME_PREFIX As String = "chk" 'The format for the the names of the checkbox controls is 'CHECKBOX_NAME_PREFIX + <name> 'where "name" is also the name of the associated Named Range Public Function CheckName_To_SectionName(ByRef strCheckName As String) CheckName_To_SectionName = Mid(strCheckName, CHECKBOX_NAME_PREFIX.Length() + 1) End Function Public Sub Show_Hide_Section(ByRef chkBox As CheckBox) ActiveSheet.Names(CheckName_To_SectionName(chkBox.Name())).RefersTo.EntireRow.Hidden = True End Sub

    Read the article

  • Cucumber could not find table; but its there. What is going on?

    - by JZ
    I'm working with cucumber and I'm running into difficulties. When I run "cucumber features", I am met with errors, cucumber is unable to find my requests table. What obvious mistake am I making? Thank you in advance! Bash: justin-zollarss-mac-pro:conversion justinz$ cucumber features Using the default profile... /Users/justinz/.gem/ruby/1.8/gems/rails-2.3.5/lib/rails/gem_dependency.rb:119:Warning: Gem::Dependency#version_requirements is deprecated and will be removed on or after August 2010. Use #requirement F-- (::) failed steps (::) Could not find table 'requests' (ActiveRecord::StatementInvalid) ./features/article_steps.rb:3 ./features/article_steps.rb:2:in `each' ./features/article_steps.rb:2:in `/^I have requests named (.+)$/' features/manage_articles.feature:7:in `Given I have requests named Foo, Bar' Failing Scenarios: cucumber features/manage_articles.feature:6 # Scenario: Conversion 1 scenario (1 failed) 3 steps (1 failed, 2 skipped) 0m0.154s justin-zollarss-mac-pro:conversion justinz$ Manage_articles.feature: Feature: Manage Articles In order to make sales As a customer I want to make conversions Scenario: Conversion Given I have requests named Foo, Bar When I go to the list of customers Then I should see a new "customer" Article_steps.rb: Given /^I have requests named (.+)$/ do |firsts| firsts.split(', ').each do |first| Request.create!(:first => first) pending # express the regexp above with the code you wish you had end end Then /^I should see a new "([^"]*)"$/ do |arg1| pending # express the regexp above with the code you wish you had end DB schema: ActiveRecord::Schema.define(:version => 20100528011731) do create_table "requests", :force => true do |t| t.string "institution" t.string "website" t.string "type" t.string "users" t.string "first" t.string "last" t.string "jobtitle" t.string "phone" t.string "email" t.datetime "created_at" t.datetime "updated_at" end end

    Read the article

  • Fix an external dependency of a ruby gem

    - by Patrick Daryll Glandien
    I am currently trying to install the gem nfoiled, which provides a ruby interface to ncurses. I do this by using gem install elliottcable-nfoiled as suggest in the README. Downloading it manually from the github repository and then installing it with rake install doesn't work because of a problem with the echoe-gem, thus I am bound to use the normal way. Unfortunately it depends on the gem ncurses-0.9.1 which is only compatible with ruby 1.8, and thus I can't install nfoiled either (since it always tries to compile ncurses-0.9.1 first): novavortex:/usr/src# gem install elliottcable-nfoiled Building native extensions. This could take a while... ... form_wrap.c: In function `rbncurs_m_new_form': form_wrap.c:395: error: `struct RArray' has no member named `len' form_wrap.c: In function `rbncurs_c_set_field_type': form_wrap.c:619: error: `struct RArray' has no member named `len' form_wrap.c: In function `rbncurs_c_set_form_fields': form_wrap.c:778: error: `struct RArray' has no member named `len' form_wrap.c: In function `make_arg': form_wrap.c:1126: error: `struct RArray' has no member named `len' make: *** [form_wrap.o] Error 1 Gem files will remain installed in /usr/local/lib/ruby/gems/1.9.1/gems/ncurses-0.9.1 for inspection. Results logged to /usr/local/lib/ruby/gems/1.9.1/gems/ncurses-0.9.1/gem_make.out novavortex:/usr/src# I managed to fix the problem in ncurses-0.9.1 (by replacing RARRAY(x)-len with RARRAY_LEN(x)) and to install it, but nfoiled still always tries to recompile it from a freshly downloaded source. How can I install nfoiled without having it recompile ncurses first?

    Read the article

  • class header+ implementation

    - by igor
    what am I doing wrong here? I keep on getting a compilation error when I try to run this in codelab (turings craft) Instructions: Write the implementation (.cpp file) of the GasTank class of the previous exercise. The full specification of the class is: A data member named amount of type double. A constructor that no parameters. The constructor initializes the data member amount to 0. A function named addGas that accepts a parameter of type double . The value of the amount instance variable is increased by the value of the parameter. A function named useGas that accepts a parameter of type double . The value of the amount data member is decreased by the value of the parameter. A function named getGasLevel that accepts no parameters. getGasLevel returns the value of the amount data member. class GasTank{ double amount; GasTank(); void addGas(double); void useGas(double); double getGasLevel();}; GasTank::GasTank(){ amount=0;} double GasTank::addGas(double a){ amount+=a;} double GasTank::useGas(double a){ amount+=a;} double GasTank::getGasLevel(){ return amount;}

    Read the article

  • define a closure as method from class

    - by user272839
    Hi, i'm trying to play with php5.3 and closure. I see here (Listing 7. Closure inside an object : http://www.ibm.com/developerworks/opensource/library/os-php-5.3new2/index.html) that it's possible to use $this in the callback function, but it's not. So I try to give $this as use variable : $self = $this; $foo = function() use($self) { //do something with $self } So to use the same example : class Dog { private $_name; protected $_color; public function __construct($name, $color) { $this->_name = $name; $this->_color = $color; } public function greet($greeting) { $self = $this; return function() use ($greeting, $self) { echo "$greeting, I am a {$self->_color} dog named {$self->_name}."; }; } } $dog = new Dog("Rover","red"); $dog->greet("Hello"); Output: Hello, I am a red dog named Rover. First of all this example does not print the string but return the function, but that's not my problem. Secondly I can't access to private or protected, because the callback function is a global function and not in the context from the Dog object. Tha't my problem. It's the same as : function greet($greeting, $object) { echo "$greeting, I am a {$self->_color} dog named {$self->_name}."; } And I want : public function greet($greeting) { echo "$greeting, I am a {$self->_color} dog named {$self->_name}."; } Which is from Dog and not global. I hope that I am clear ...

    Read the article

  • how to insert excel-2003 values into SQL2005 database?

    - by vas
    Are there any rules / guidelines for DATA form XLS sheets to be inserted into SQL- DB? I have a group of Excel templates in 2005.Each concerned cell in Excel template is named. When Excel sheets are filled, saved and submitted , the values are transferred to the database. Excel sheets have names for various cells that are to b e filled by the user EX:- for the total number of Milk in the Beginning a given month , there is an Excel Cell Named "mtsBpiPTR180" Total number of Milk in the Ending a given month , there is an Excel Cell Named **"mtsEpiPTR180"** I have added 2 new cells , named "mtsBpiPTR180PA" and "mtsEpiPTR180PA". Now I try to upload the Excel File. But I AM UNABLE TO SEE MY FILLED DATA FROM "mtsBpiPTR180PA" and "mtsEpiPTR180PA" INTO THE RELATED DB/table. The above 2 are empty in the DB/table, even though I have filled them and successfully filed the Excel sheets Now no matter how much I search in the DB/stored procs i am unable to the ACTUAL STORED PROC or how the Data form Excel sheet is inserted into Tables WHERE DATA FROM XLS is inserted into DB. So was wondering:- Are there any rules / guidelines for DATA form XLS sheets to be inserted into SQL- DB?

    Read the article

  • how to create following Java applicatin? [on hold]

    - by Tushar Bichwe
    Write a JAVA program which performs the following listed operations: A. Create a package named MyEmpPackage which consists of following classes A class named Employee which stores information like the Emp number, first name, middle name, last name, address, designation and salary. The class should also contain appropriate get and set methods. 05 A class named AddEmployeeFrame which displays a frame consisting of appropriate controls to enter the details of a Employee and store these details in the Employee class object. The frame should also have three buttons with the caption as “Add Record” and “Delete Record” and “Exit”. 10 A class named MyCustomListener which should work as a user – defined event listener to handle required events as mentioned in following points. 05 B When the “Add Record” button is clicked, the dialog box should be appeared with asking the user “Do you really want to add record in the file”. If the user selects Yes than the record should be saved in the file. 10 When the “Exit” button is clicked, the frame should be closed. 10 [Note: Use the MyCustomListener class only to handle the appropriate events] C The “Delete Record” button should open a new frame which should take input of delete criteria using a radio button. The radio button should provide facility to delete on basis of first name, middle name or last name. 10 The new frame should also have a text box to input the delete criteria value. 10 The record should be deleted from the file and a message dialog should appear with the message that “Record is successfully Deleted”. 10 [Note: Use the MyCustomListener class only to handle the appropriate events] D Provide proper error messages and perform appropriate exceptions where ever required in all the classes 10

    Read the article

  • Preserving Permalinks

    - by Daniel Moth
    One of the things that gets me on a rant is websites that break permalinks. If you have posted something somewhere and there is a public URL pointing to it, that URL should never ever return a 404. You are breaking all websites that ever linked to you and you are breaking all search engine links to your content (that others will try and follow). It is a pet peeve of mine. So when I had to move my blog, obviously I would preserve the root URL (www.danielmoth.com/Blog/), but I also wanted to preserve every URL my blog has generated over the years. To be clear, our focus here is on the URL formatting, not the content migration which I'll talk about in my next post. In this post, I'll describe my solution first and then what it solves. 1. The IIS7 Rewrite Module and web.config There are a few ways you can map an old URL to a new one (so when requests to the old URL come in, they get redirected to the new one). The new blog engine I use (dasBlog) has built-in functionality to do that (Scott refers to it here). Instead, the way I chose to address the issue was to use the IIS7 rewrite module. The IIS7 rewrite module allows redirecting URLs based on pattern matching, regular expressions and, of course, hardcoded full URLs for things that don't fall into any pattern. You can configure it visually from IIS Manager using a handy dialog that allows testing patterns against input URLs. Here is what mine looked like after configuring a few rules: To learn more about this technology check out this video, the reference page and this overview blog post; all 3 pages have a collection of related resources at the bottom worth checking out too. All the visual configuration ends up in a web.config file at the root folder of your website. If you are on a shared hosting service, probably the only way you can use the Rewrite Module is by directly editing the web.config file. Next, I'll describe the URLs I had to map and how that manifested itself in the web.config file. What I did was create the rules locally using the GUI, and then took the generated web.config file and uploaded it to my live site. You can view my web.config here. 2. Monthly Archives Observe the difference between the way the two blog engines generate this type of URL Blogger: /Blog/2004_07_01_mothblog_archive.html dasBlog: /Blog/default,month,2004-07.aspx In my web.config file, the rule that deals with this is the one named "monthlyarchive_redirect". 3. Categories Observe the difference between the way the two blog engines generate this type of URL Blogger: /Blog/labels/Personal.html dasBlog: /Blog/CategoryView,category,Personal.aspx In my web.config file the rule that deals with this is the one named "category_redirect". 4. Posts Observe the difference between the way the two blog engines generate this type of URL Blogger: /Blog/2004/07/hello-world.html dasBlog: /Blog/Hello-World.aspx In my web.config file the rule that deals with this is the one named "post_redirect". Note: The decision is taken to use dasBlog URLs that do not include the date info (see the description of my Appearance settings). If we included the date info then it would have to include the day part, which blogger did not generate. This makes it impossible to redirect correctly and to have a single permalink for blog posts moving forward. An implication of this decision, is that no two blog posts can have the same title. The tool I will describe in my next post (inelegantly) deals with duplicates, but not with triplicates or higher. 5. Unhandled by a generic rule Unfortunately, the two blog engines use different rules for generating URLs for blog posts. Most of the time the conversion is as simple as the example of the previous section where a post titled "Hello World" generates a URL with the words separated by a hyphen. Some times that is not the case, for example: /Blog/2006/05/medc-wrap-up.html /Blog/MEDC-Wrapup.aspx or /Blog/2005/01/best-of-moth-2004.html /Blog/Best-Of-The-Moth-2004.aspx or /Blog/2004/11/more-windows-mobile-2005-details.html /Blog/More-Windows-Mobile-2005-Details-Emerge.aspx In short, blogger does not add words to the title beyond ~39 characters, it drops some words from the title generation (e.g. a, an, on, the), and it preserve hyphens that appear in the title. For this reason, we need to detect these and explicitly list them for redirects (no regular expression can help here because the full set of rules is not listed anywhere). In my web.config file the rule that deals with this is the one named "Redirect rule1 for FullRedirects" combined with the rewriteMap named "StaticRedirects". Note: The tool I describe in my next post will detect all the URLs that need to be explicitly redirected and will list them in a file ready for you to copy them to your web.config rewriteMap. 6. C# code doing the same as the web.config I wrote some naive code that does the same thing as the web.config: given a string it will return a new string converted according to the 3 rules above. It does not take into account the 4th case where an explicit hard-coded conversion is needed (the tool I present in the next post does take that into account). static string REGEX_post_redirect = "[0-9]{4}/[0-9]{2}/([0-9a-z-]+).html"; static string REGEX_category_redirect = "labels/([_0-9a-z-% ]+).html"; static string REGEX_monthlyarchive_redirect = "([0-9]{4})_([0-9]{2})_[0-9]{2}_mothblog_archive.html"; static string Redirect(string oldUrl) { GroupCollection g; if (RunRegExOnIt(oldUrl, REGEX_post_redirect, 2, out g)) return string.Concat(g[1].Value, ".aspx"); if (RunRegExOnIt(oldUrl, REGEX_category_redirect, 2, out g)) return string.Concat("CategoryView,category,", g[1].Value, ".aspx"); if (RunRegExOnIt(oldUrl, REGEX_monthlyarchive_redirect, 3, out g)) return string.Concat("default,month,", g[1].Value, "-", g[2], ".aspx"); return string.Empty; } static bool RunRegExOnIt(string toRegEx, string pattern, int groupCount, out GroupCollection g) { if (pattern.Length == 0) { g = null; return false; } g = new Regex(pattern, RegexOptions.IgnoreCase | RegexOptions.Compiled).Match(toRegEx).Groups; return (g.Count == groupCount); } Comments about this post welcome at the original blog.

    Read the article

  • C#: Optional Parameters - Pros and Pitfalls

    - by James Michael Hare
    When Microsoft rolled out Visual Studio 2010 with C# 4, I was very excited to learn how I could apply all the new features and enhancements to help make me and my team more productive developers. Default parameters have been around forever in C++, and were intentionally omitted in Java in favor of using overloading to satisfy that need as it was though that having too many default parameters could introduce code safety issues.  To some extent I can understand that move, as I’ve been bitten by default parameter pitfalls before, but at the same time I feel like Java threw out the baby with the bathwater in that move and I’m glad to see C# now has them. This post briefly discusses the pros and pitfalls of using default parameters.  I’m avoiding saying cons, because I really don’t believe using default parameters is a negative thing, I just think there are things you must watch for and guard against to avoid abuses that can cause code safety issues. Pro: Default Parameters Can Simplify Code Let’s start out with positives.  Consider how much cleaner it is to reduce all the overloads in methods or constructors that simply exist to give the semblance of optional parameters.  For example, we could have a Message class defined which allows for all possible initializations of a Message: 1: public class Message 2: { 3: // can either cascade these like this or duplicate the defaults (which can introduce risk) 4: public Message() 5: : this(string.Empty) 6: { 7: } 8:  9: public Message(string text) 10: : this(text, null) 11: { 12: } 13:  14: public Message(string text, IDictionary<string, string> properties) 15: : this(text, properties, -1) 16: { 17: } 18:  19: public Message(string text, IDictionary<string, string> properties, long timeToLive) 20: { 21: // ... 22: } 23: }   Now consider the same code with default parameters: 1: public class Message 2: { 3: // can either cascade these like this or duplicate the defaults (which can introduce risk) 4: public Message(string text = "", IDictionary<string, string> properties = null, long timeToLive = -1) 5: { 6: // ... 7: } 8: }   Much more clean and concise and no repetitive coding!  In addition, in the past if you wanted to be able to cleanly supply timeToLive and accept the default on text and properties above, you would need to either create another overload, or pass in the defaults explicitly.  With named parameters, though, we can do this easily: 1: var msg = new Message(timeToLive: 100);   Pro: Named Parameters can Improve Readability I must say one of my favorite things with the default parameters addition in C# is the named parameters.  It lets code be a lot easier to understand visually with no comments.  Think how many times you’ve run across a TimeSpan declaration with 4 arguments and wondered if they were passing in days/hours/minutes/seconds or hours/minutes/seconds/milliseconds.  A novice running through your code may wonder what it is.  Named arguments can help resolve the visual ambiguity: 1: // is this days/hours/minutes/seconds (no) or hours/minutes/seconds/milliseconds (yes) 2: var ts = new TimeSpan(1, 2, 3, 4); 3:  4: // this however is visually very explicit 5: var ts = new TimeSpan(days: 1, hours: 2, minutes: 3, seconds: 4);   Or think of the times you’ve run across something passing a Boolean literal and wondered what it was: 1: // what is false here? 2: var sub = CreateSubscriber(hostname, port, false); 3:  4: // aha! Much more visibly clear 5: var sub = CreateSubscriber(hostname, port, isBuffered: false);   Pitfall: Don't Insert new Default Parameters In Between Existing Defaults Now let’s consider a two potential pitfalls.  The first is really an abuse.  It’s not really a fault of the default parameters themselves, but a fault in the use of them.  Let’s consider that Message constructor again with defaults.  Let’s say you want to add a messagePriority to the message and you think this is more important than a timeToLive value, so you decide to put messagePriority before it in the default, this gives you: 1: public class Message 2: { 3: public Message(string text = "", IDictionary<string, string> properties = null, int priority = 5, long timeToLive = -1) 4: { 5: // ... 6: } 7: }   Oh boy have we set ourselves up for failure!  Why?  Think of all the code out there that could already be using the library that already specified the timeToLive, such as this possible call: 1: var msg = new Message(“An error occurred”, myProperties, 1000);   Before this specified a message with a TTL of 1000, now it specifies a message with a priority of 1000 and a time to live of -1 (infinite).  All of this with NO compiler errors or warnings. So the rule to take away is if you are adding new default parameters to a method that’s currently in use, make sure you add them to the end of the list or create a brand new method or overload. Pitfall: Beware of Default Parameters in Inheritance and Interface Implementation Now, the second potential pitfalls has to do with inheritance and interface implementation.  I’ll illustrate with a puzzle: 1: public interface ITag 2: { 3: void WriteTag(string tagName = "ITag"); 4: } 5:  6: public class BaseTag : ITag 7: { 8: public virtual void WriteTag(string tagName = "BaseTag") { Console.WriteLine(tagName); } 9: } 10:  11: public class SubTag : BaseTag 12: { 13: public override void WriteTag(string tagName = "SubTag") { Console.WriteLine(tagName); } 14: } 15:  16: public static class Program 17: { 18: public static void Main() 19: { 20: SubTag subTag = new SubTag(); 21: BaseTag subByBaseTag = subTag; 22: ITag subByInterfaceTag = subTag; 23:  24: // what happens here? 25: subTag.WriteTag(); 26: subByBaseTag.WriteTag(); 27: subByInterfaceTag.WriteTag(); 28: } 29: }   What happens?  Well, even though the object in each case is SubTag whose tag is “SubTag”, you will get: 1: SubTag 2: BaseTag 3: ITag   Why?  Because default parameter are resolved at compile time, not runtime!  This means that the default does not belong to the object being called, but by the reference type it’s being called through.  Since the SubTag instance is being called through an ITag reference, it will use the default specified in ITag. So the moral of the story here is to be very careful how you specify defaults in interfaces or inheritance hierarchies.  I would suggest avoiding repeating them, and instead concentrating on the layer of classes or interfaces you must likely expect your caller to be calling from. For example, if you have a messaging factory that returns an IMessage which can be either an MsmqMessage or JmsMessage, it only makes since to put the defaults at the IMessage level since chances are your user will be using the interface only. So let’s sum up.  In general, I really love default and named parameters in C# 4.0.  I think they’re a great tool to help make your code easier to read and maintain when used correctly. On the plus side, default parameters: Reduce redundant overloading for the sake of providing optional calling structures. Improve readability by being able to name an ambiguous argument. But remember to make sure you: Do not insert new default parameters in the middle of an existing set of default parameters, this may cause unpredictable behavior that may not necessarily throw a syntax error – add to end of list or create new method. Be extremely careful how you use default parameters in inheritance hierarchies and interfaces – choose the most appropriate level to add the defaults based on expected usage. Technorati Tags: C#,.NET,Software,Default Parameters

    Read the article

  • Portal And Content - Content Integration - Best Practices

    - by Stefan Krantz
    Lately we have seen an increase in projects that have failed to either get user friendly content integration or non satisfactory performance. Our intention is to mitigate any knowledge gap that our previous post might have left you with, therefore this post will repeat some recommendation or reference back to old useful post. Moreover this post will help you understand ground up how to design, architect and implement business enabled, responsive and performing portals with complex requirements on business centric information publishing. Design the Information Model The key to successful portal deployments is Information modeling, it's a key task to understand the use case you designing for, therefore I have designed a set of question you need to ask yourself or your customer: Question: Who will own the content, IT or Business? Answer: BusinessQuestion: Who will publish the content, IT or Business? Answer: BusinessQuestion: Will there be multiple publishers? Answer: YesQuestion: Are the publishers computer scientist?Answer: NoQuestion: How often do the information changes, daily, weekly, monthly?Answer: Daily, weekly If your answers to the questions matches at least 2, we strongly recommend you design your content with following principles: Divide your pages in to logical sections, where each section is marked with its purpose Assign capabilities to each section, does it contain text, images, formatting and/or is it static and is populated through other contextual information Select editor/design element type WYSIWYG - Rich Text Plain Text - non-format text Image - Image object Static List - static list of formatted informationDynamic Data List - assembled information from multiple data files through CMIS query The result of such design map could look like following below examples: Based on the outcome of the required elements in the design column 3 from the left you will now simply design a data model in WebCenter Content - Site Studio by creating a Region Definition structure matching your design requirements.For more information on how to create a Region definition see following post: Region Definition Post - note see instruction 7 for details. Each region definition can now be used to instantiate data files, a data file will hold the actual data for each element in the region definition. Another way you can see this is to compare the region definition as an extension to the metadata model in WebCenter Content for each data file item. Design content templates With a solid dependable information model we can now proceed to template creation and page design, in this phase focuses on how to place the content sections from the region definition on the page via a Content Presenter template. Remember by creating content presenter templates you will leverage the latest and most integrated technology WebCenter has to offer. This phase is much easier since the you already have the information model and design wire-frames to base the logic on, however there is still few considerations to pay attention to: Base the template on ADF and make only necessary exceptions to markup when required Leverage ADF design components for Tabs, Accordions and other similar components, this way the design in the content published areas will comply with other design areas based on custom ADF taskflows There is no performance impact when using meta data or region definition based data All data access regardless of type, metadata or xml data it can be accessed via the Content Presenter - Node. See below for applied examples on how to access data Access metadata property from Document - #{node.propertyMap['myProp'].value}myProp in this example can be for instance (dDocName, dDocTitle, xComments or any other available metadata) Access element data from data file xml - #{node.propertyMap['[Region Definition Name]:[Element name]'].asTextHtml}Region Definition Name is the expect region definition that the current data file is instantiatingElement name is the element value you like to grab from the data file I recommend you read following  useful post on content template topic:CMIS queries and template creation - note see instruction 9 for detailsStatic List template rendering For more information on templates:Single Item Content TemplateMulti Item Content TemplateExpression Language Internationalization Considerations When integrating content assets via content presenter you by now probably understand that the content item/data file is wired to the page, what is also pretty common at this stage is that the content item/data file only support one language since its not practical or business friendly to mix that into a complex structure. Therefore you will be left with a very common dilemma that you will have to either build a complete new portal for each locale, which is not an good option! However with little bit of information modeling and clear naming convention this can be addressed. Basically you can simply make sure that all content item/data file are named with a predictable naming convention like "Content1_EN" for the English rendition and "Content1_ES" for the Spanish rendition. This way through simple none complex customizations you will be able to dynamically switch the actual content item/data file just before rendering. By following proposed approach above you not only enable a simple mechanism for internationalized content you also preserve the functionality in the content presenter to support business accessible run-time publishing of information on existing and new pages. I recommend you read following useful post on Internationalization topics:Internationalize with Content Presenter Integrate with Review & Approval processes Today the Review and approval functionality and configuration is based out of WebCenter Content - Criteria Workflows. Criteria Workflows uses the metadata of the checked in document to evaluate if the document is under any review/approval process. So for instance if a Criteria Workflow is configured to force any documents with Version = "2" or "higher" and Content Type is "Instructions", any matching content item version on check in will now enter the workflow before getting released for general access. Few things to consider when configuring Criteria Workflows: Make sure to not trigger on version one for Content Items that are Data Files - if you trigger on version 1 you will not only approve an empty document you will also have a content presenter pointing to a none existing document - since the document will only be available after successful completion of the workflow Approval workflows sometimes requires more complex criteria, the recommendation if that is the case is that the meta data triggering such criteria is automatically populated, this can be achieved through many approaches including Content Profiles Criteria workflows are configured and managed in WebCenter Content Administration Applets where you can configure one or more workflows. When you configured Criteria workflows the Content Presenter will support the editors with the approval process directly inline in the "Contribution mode" of the portal. In addition to approve/reject and details of the task, the content presenter natively support the user to view the current and future version of the change he/she is approving. See below for example: Architectural recommendation To support review&approval processes - minimize the amount of data files per page Each CMIS query can consume significant time depending on the complexity of the query - minimize the amount of CMIS queries per page Use Content Presenter Templates based on ADF - this way you minimize the design considerations and optimize the usage of caching Implement the page in as few Data files as possible - simplifies publishing process, increases performance and simplifies release process Named data file (node) or list of named nodes when integrating to pages increases performance vs. querying for data Named data file (node) or list of named nodes when integrating to pages enables business centric page creation and publishing and reduces the need for IT department interaction Summary Just because one architectural decision solves a business problem it doesn't mean its the right one, when designing portals all architecture has to be in harmony and not impacting each other. For instance the most technical complex solution is not always the best since it will most likely defeat the business accessibility, performance or both, therefore the best approach is to first design for simplicity that even a non-technical user can operate, after that consider the performance impact and final look at the technology challenges these brings and workaround them first with out-of-the-box features, after that design and develop functions to complement the short comings.

    Read the article

  • Writing Unit Tests for an ASP.NET MVC Action Method that handles Ajax Request and Normal Request

    - by shiju
    In this blog post, I will demonstrate how to write unit tests for an ASP.NET MVC action method, which handles both Ajax request and normal HTTP Request. I will write a unit test for specifying the behavior of an Ajax request and will write another unit test for specifying the behavior of a normal HTTP request. Both Ajax request and normal request will be handled by a single action method. So the ASP.NET MVC action method will be execute HTTP Request object’s IsAjaxRequest method for identifying whether it is an Ajax request or not. So we have to create mock object for Request object and also have to make as a Ajax request from the unit test for verifying the behavior of an Ajax request. I have used NUnit and Moq for writing unit tests. Let me write a unit test for a Ajax request Code Snippet [Test] public void Index_AjaxRequest_Returns_Partial_With_Expense_List() {     // Arrange       Mock<HttpRequestBase> request = new Mock<HttpRequestBase>();     Mock<HttpResponseBase> response = new Mock<HttpResponseBase>();     Mock<HttpContextBase> context = new Mock<HttpContextBase>();       context.Setup(c => c.Request).Returns(request.Object);     context.Setup(c => c.Response).Returns(response.Object);     //Add XMLHttpRequest request header     request.Setup(req => req["X-Requested-With"]).         Returns("XMLHttpRequest");       IEnumerable<Expense> fakeExpenses = GetMockExpenses();     expenseRepository.Setup(x => x.GetMany(It.         IsAny<Expression<Func<Expense, bool>>>())).         Returns(fakeExpenses);     ExpenseController controller = new ExpenseController(         commandBus.Object, categoryRepository.Object,         expenseRepository.Object);     controller.ControllerContext = new ControllerContext(         context.Object, new RouteData(), controller);     // Act     var result = controller.Index(null, null) as PartialViewResult;     // Assert     Assert.AreEqual("_ExpenseList", result.ViewName);     Assert.IsNotNull(result, "View Result is null");     Assert.IsInstanceOf(typeof(IEnumerable<Expense>),             result.ViewData.Model, "Wrong View Model");     var expenses = result.ViewData.Model as IEnumerable<Expense>;     Assert.AreEqual(3, expenses.Count(),         "Got wrong number of Categories");         }   In the above unit test, we are calling Index action method of a controller named ExpenseController, which will returns a PartialView named _ExpenseList, if it is an Ajax request. We have created mock object for HTTPContextBase and setup XMLHttpRequest request header for Request object’s X-Requested-With for making it as a Ajax request. We have specified the ControllerContext property of the controller with mocked object HTTPContextBase. Code Snippet controller.ControllerContext = new ControllerContext(         context.Object, new RouteData(), controller); Let me write a unit test for a normal HTTP method Code Snippet [Test] public void Index_NormalRequest_Returns_Index_With_Expense_List() {     // Arrange               Mock<HttpRequestBase> request = new Mock<HttpRequestBase>();     Mock<HttpResponseBase> response = new Mock<HttpResponseBase>();     Mock<HttpContextBase> context = new Mock<HttpContextBase>();       context.Setup(c => c.Request).Returns(request.Object);     context.Setup(c => c.Response).Returns(response.Object);       IEnumerable<Expense> fakeExpenses = GetMockExpenses();       expenseRepository.Setup(x => x.GetMany(It.         IsAny<Expression<Func<Expense, bool>>>())).         Returns(fakeExpenses);     ExpenseController controller = new ExpenseController(         commandBus.Object, categoryRepository.Object,         expenseRepository.Object);     controller.ControllerContext = new ControllerContext(         context.Object, new RouteData(), controller);     // Act     var result = controller.Index(null, null) as ViewResult;     // Assert     Assert.AreEqual("Index", result.ViewName);     Assert.IsNotNull(result, "View Result is null");     Assert.IsInstanceOf(typeof(IEnumerable<Expense>),             result.ViewData.Model, "Wrong View Model");     var expenses = result.ViewData.Model         as IEnumerable<Expense>;     Assert.AreEqual(3, expenses.Count(),         "Got wrong number of Categories"); }   In the above unit test, we are not specifying the XMLHttpRequest request header for Request object’s X-Requested-With, so that it will be normal HTTP Request. If this is a normal request, the action method will return a ViewResult with a view template named Index. The below is the implementation of Index action method Code Snippet public ActionResult Index(DateTime? startDate, DateTime? endDate) {     //If date is not passed, take current month's first and last date     DateTime dtNow;     dtNow = DateTime.Today;     if (!startDate.HasValue)     {         startDate = new DateTime(dtNow.Year, dtNow.Month, 1);         endDate = startDate.Value.AddMonths(1).AddDays(-1);     }     //take last date of start date's month, if end date is not passed     if (startDate.HasValue && !endDate.HasValue)     {         endDate = (new DateTime(startDate.Value.Year,             startDate.Value.Month, 1)).AddMonths(1).AddDays(-1);     }     var expenses = expenseRepository.GetMany(         exp => exp.Date >= startDate && exp.Date <= endDate);     //if request is Ajax will return partial view     if (Request.IsAjaxRequest())     {         return PartialView("_ExpenseList", expenses);     }     //set start date and end date to ViewBag dictionary     ViewBag.StartDate = startDate.Value.ToShortDateString();     ViewBag.EndDate = endDate.Value.ToShortDateString();     //if request is not ajax     return View("Index",expenses); }   The index action method will returns a PartialView named _ExpenseList, if it is an Ajax request and will returns a View named Index if it is a normal request. Source Code The source code has been taken from my EFMVC app which can download from here

    Read the article

  • jQuery Globalization Plugin from Microsoft

    - by ScottGu
    Last month I blogged about how Microsoft is starting to make code contributions to jQuery, and about some of the first code contributions we were working on: jQuery Templates and Data Linking support. Today, we released a prototype of a new jQuery Globalization Plugin that enables you to add globalization support to your JavaScript applications. This plugin includes globalization information for over 350 cultures ranging from Scottish Gaelic, Frisian, Hungarian, Japanese, to Canadian English.  We will be releasing this plugin to the community as open-source. You can download our prototype for the jQuery Globalization plugin from our Github repository: http://github.com/nje/jquery-glob You can also download a set of samples that demonstrate some simple use-cases with it here. Understanding Globalization The jQuery Globalization plugin enables you to easily parse and format numbers, currencies, and dates for different cultures in JavaScript. For example, you can use the Globalization plugin to display the proper currency symbol for a culture: You also can use the Globalization plugin to format dates so that the day and month appear in the right order and the day and month names are correctly translated: Notice above how the Arabic year is displayed as 1431. This is because the year has been converted to use the Arabic calendar. Some cultural differences, such as different currency or different month names, are obvious. Other cultural differences are surprising and subtle. For example, in some cultures, the grouping of numbers is done unevenly. In the "te-IN" culture (Telugu in India), groups have 3 digits and then 2 digits. The number 1000000 (one million) is written as "10,00,000". Some cultures do not group numbers at all. All of these subtle cultural differences are handled by the jQuery Globalization plugin automatically. Getting dates right can be especially tricky. Different cultures have different calendars such as the Gregorian and UmAlQura calendars. A single culture can even have multiple calendars. For example, the Japanese culture uses both the Gregorian calendar and a Japanese calendar that has eras named after Japanese emperors. The Globalization Plugin includes methods for converting dates between all of these different calendars. Using Language Tags The jQuery Globalization plugin uses the language tags defined in the RFC 4646 and RFC 5646 standards to identity cultures (see http://tools.ietf.org/html/rfc5646). A language tag is composed out of one or more subtags separated by hyphens. For example: Language Tag Language Name (in English) en-AU English (Australia) en-BZ English (Belize) en-CA English (Canada) Id Indonesian zh-CHS Chinese (Simplified) Legacy Zu isiZulu Notice that a single language, such as English, can have several language tags. Speakers of English in Canada format numbers, currencies, and dates using different conventions than speakers of English in Australia or the United States. You can find the language tag for a particular culture by using the Language Subtag Lookup tool located here:  http://rishida.net/utils/subtags/ The jQuery Globalization plugin download includes a folder named globinfo that contains the information for each of the 350 cultures. Actually, this folder contains more than 700 files because the folder includes both minified and un-minified versions of each file. For example, the globinfo folder includes JavaScript files named jQuery.glob.en-AU.js for English Australia, jQuery.glob.id.js for Indonesia, and jQuery.glob.zh-CHS for Chinese (Simplified) Legacy. Example: Setting a Particular Culture Imagine that you have been asked to create a German website and want to format all of the dates, currencies, and numbers using German formatting conventions correctly in JavaScript on the client. The HTML for the page might look like this: Notice the span tags above. They mark the areas of the page that we want to format with the Globalization plugin. We want to format the product price, the date the product is available, and the units of the product in stock. To use the jQuery Globalization plugin, we’ll add three JavaScript files to the page: the jQuery library, the jQuery Globalization plugin, and the culture information for a particular language: In this case, I’ve statically added the jQuery.glob.de-DE.js JavaScript file that contains the culture information for German. The language tag “de-DE” is used for German as spoken in Germany. Now that I have all of the necessary scripts, I can use the Globalization plugin to format the product price, date available, and units in stock values using the following client-side JavaScript: The jQuery Globalization plugin extends the jQuery library with new methods - including new methods named preferCulture() and format(). The preferCulture() method enables you to set the default culture used by the jQuery Globalization plugin methods. Notice that the preferCulture() method accepts a language tag. The method will find the closest culture that matches the language tag. The $.format() method is used to actually format the currencies, dates, and numbers. The second parameter passed to the $.format() method is a format specifier. For example, passing “c” causes the value to be formatted as a currency. The ReadMe file at github details the meaning of all of the various format specifiers: http://github.com/nje/jquery-glob When we open the page in a browser, everything is formatted correctly according to German language conventions. A euro symbol is used for the currency symbol. The date is formatted using German day and month names. Finally, a period instead of a comma is used a number separator: You can see a running example of the above approach with the 3_GermanSite.htm file in this samples download. Example: Enabling a User to Dynamically Select a Culture In the previous example we explicitly said that we wanted to globalize in German (by referencing the jQuery.glob.de-DE.js file). Let’s now look at the first of a few examples that demonstrate how to dynamically set the globalization culture to use. Imagine that you want to display a dropdown list of all of the 350 cultures in a page. When someone selects a culture from the dropdown list, you want all of the dates in the page to be formatted using the selected culture. Here’s the HTML for the page: Notice that all of the dates are contained in a <span> tag with a data-date attribute (data-* attributes are a new feature of HTML 5 that conveniently also still work with older browsers). We’ll format the date represented by the data-date attribute when a user selects a culture from the dropdown list. In order to display dates for any possible culture, we’ll include the jQuery.glob.all.js file like this: The jQuery Globalization plugin includes a JavaScript file named jQuery.glob.all.js. This file contains globalization information for all of the more than 350 cultures supported by the Globalization plugin.  At 367KB minified, this file is not small. Because of the size of this file, unless you really need to use all of these cultures at the same time, we recommend that you add the individual JavaScript files for particular cultures that you intend to support instead of the combined jQuery.glob.all.js to a page. In the next sample I’ll show how to dynamically load just the language files you need. Next, we’ll populate the dropdown list with all of the available cultures. We can use the $.cultures property to get all of the loaded cultures: Finally, we’ll write jQuery code that grabs every span element with a data-date attribute and format the date: The jQuery Globalization plugin’s parseDate() method is used to convert a string representation of a date into a JavaScript date. The plugin’s format() method is used to format the date. The “D” format specifier causes the date to be formatted using the long date format. And now the content will be globalized correctly regardless of which of the 350 languages a user visiting the page selects.  You can see a running example of the above approach with the 4_SelectCulture.htm file in this samples download. Example: Loading Globalization Files Dynamically As mentioned in the previous section, you should avoid adding the jQuery.glob.all.js file to a page whenever possible because the file is so large. A better alternative is to load the globalization information that you need dynamically. For example, imagine that you have created a dropdown list that displays a list of languages: The following jQuery code executes whenever a user selects a new language from the dropdown list. The code checks whether the globalization file associated with the selected language has already been loaded. If the globalization file has not been loaded then the globalization file is loaded dynamically by taking advantage of the jQuery $.getScript() method. The globalizePage() method is called after the requested globalization file has been loaded, and contains the client-side code to perform the globalization. The advantage of this approach is that it enables you to avoid loading the entire jQuery.glob.all.js file. Instead you only need to load the files that you need and you don’t need to load the files more than once. The 5_Dynamic.htm file in this samples download demonstrates how to implement this approach. Example: Setting the User Preferred Language Automatically Many websites detect a user’s preferred language from their browser settings and automatically use it when globalizing content. A user can set a preferred language for their browser. Then, whenever the user requests a page, this language preference is included in the request in the Accept-Language header. When using Microsoft Internet Explorer, you can set your preferred language by following these steps: Select the menu option Tools, Internet Options. Select the General tab. Click the Languages button in the Appearance section. Click the Add button to add a new language to the list of languages. Move your preferred language to the top of the list. Notice that you can list multiple languages in the Language Preference dialog. All of these languages are sent in the order that you listed them in the Accept-Language header: Accept-Language: fr-FR,id-ID;q=0.7,en-US;q=0.3 Strangely, you cannot retrieve the value of the Accept-Language header from client JavaScript. Microsoft Internet Explorer and Mozilla Firefox support a bevy of language related properties exposed by the window.navigator object, such as windows.navigator.browserLanguage and window.navigator.language, but these properties represent either the language set for the operating system or the language edition of the browser. These properties don’t enable you to retrieve the language that the user set as his or her preferred language. The only reliable way to get a user’s preferred language (the value of the Accept-Language header) is to write server code. For example, the following ASP.NET page takes advantage of the server Request.UserLanguages property to assign the user’s preferred language to a client JavaScript variable named acceptLanguage (which then allows you to access the value using client-side JavaScript): In order for this code to work, the culture information associated with the value of acceptLanguage must be included in the page. For example, if someone’s preferred culture is fr-FR (French in France) then you need to include either the jQuery.glob.fr-FR.js or the jQuery.glob.all.js JavaScript file in the page or the culture information won’t be available.  The “6_AcceptLanguages.aspx” sample in this samples download demonstrates how to implement this approach. If the culture information for the user’s preferred language is not included in the page then the $.preferCulture() method will fall back to using the neutral culture (for example, using jQuery.glob.fr.js instead of jQuery.glob.fr-FR.js). If the neutral culture information is not available then the $.preferCulture() method falls back to the default culture (English). Example: Using the Globalization Plugin with the jQuery UI DatePicker One of the goals of the Globalization plugin is to make it easier to build jQuery widgets that can be used with different cultures. We wanted to make sure that the jQuery Globalization plugin could work with existing jQuery UI plugins such as the DatePicker plugin. To that end, we created a patched version of the DatePicker plugin that can take advantage of the Globalization plugin when rendering a calendar. For example, the following figure illustrates what happens when you add the jQuery Globalization and the patched jQuery UI DatePicker plugin to a page and select Indonesian as the preferred culture: Notice that the headers for the days of the week are displayed using Indonesian day name abbreviations. Furthermore, the month names are displayed in Indonesian. You can download the patched version of the jQuery UI DatePicker from our github website. Or you can use the version included in this samples download and used by the 7_DatePicker.htm sample file. Summary I’m excited about our continuing participation in the jQuery community. This Globalization plugin is the third jQuery plugin that we’ve released. We’ve really appreciated all of the great feedback and design suggestions on the jQuery templating and data-linking prototypes that we released earlier this year.  We also want to thank the jQuery and jQuery UI teams for working with us to create these plugins. Hope this helps, Scott P.S. In addition to blogging, I am also now using Twitter for quick updates and to share links. You can follow me at: twitter.com/scottgu

    Read the article

  • Windows Azure Evolution &ndash; Caching (Preview)

    - by Shaun
    Caching is a popular topic when we are building a high performance and high scalable system not only on top of the cloud platform but the on-premise environment as well. On March 2011 the Windows Azure AppFabric Caching had been production launched. It provides an in-memory, distributed caching service over the cloud. And now, in this June 2012 update, the cache team announce a grand new caching solution on Windows Azure, which is called Windows Azure Caching (Preview). And the original Windows Azure AppFabric Caching was renamed to Windows Azure Shared Caching.   What’s Caching (Preview) If you had been using the Shared Caching you should know that it is constructed by a bunch of cache servers. And when you want to use you should firstly create a cache account from the developer portal and specify the size you want to use, which means how much memory you can use to store your data that wanted to be cached. Then you can add, get and remove them through your code through the cache URL. The Shared Caching is a multi-tenancy system which host all cached items across all users. So you don’t know which server your data was located. This caching mode works well and can take most of the cases. But it has some problems. The first one is the performance. Since the Shared Caching is a multi-tenancy system, which means all cache operations should go through the Shared Caching gateway and then routed to the server which have the data your are looking for. Even though there are some caches in the Shared Caching system it also takes time from your cloud services to the cache service. Secondary, the Shared Caching service works as a block box to the developer. The only thing we know is my cache endpoint, and that’s all. Someone may satisfied since they don’t want to care about anything underlying. But if you need to know more and want more control that’s impossible in the Shared Caching. The last problem would be the price and cost-efficiency. You pay the bill based on how much cache you requested per month. But when we host a web role or worker role, it seldom consumes all of the memory and CPU in the virtual machine (service instance). If using Shared Caching we have to pay for the cache service while waste of some of our memory and CPU locally. Since the issues above Microsoft offered a new caching mode over to us, which is the Caching (Preview). Instead of having a separated cache service, the Caching (Preview) leverage the memory and CPU in our cloud services (web role and worker role) as the cache clusters. Hence the Caching (Preview) runs on the virtual machines which hosted or near our cloud applications. Without any gateway and routing, since it located in the same data center and same racks, it provides really high performance than the Shared Caching. The Caching (Preview) works side-by-side to our application, initialized and worked as a Windows Service running in the virtual machines invoked by the startup tasks from our roles, we could get more information and control to them. And since the Caching (Preview) utilizes the memory and CPU from our existing cloud services, so it’s free. What we need to pay is the original computing price. And the resource on each machines could be used more efficiently.   Enable Caching (Preview) It’s very simple to enable the Caching (Preview) in a cloud service. Let’s create a new windows azure cloud project from Visual Studio and added an ASP.NET Web Role. Then open the role setting and select the Caching page. This is where we enable and configure the Caching (Preview) on a role. To enable the Caching (Preview) just open the “Enable Caching (Preview Release)” check box. And then we need to specify which mode of the caching clusters we want to use. There are two kinds of caching mode, co-located and dedicate. The co-located mode means we use the memory in the instances we run our cloud services (web role or worker role). By using this mode we must specify how many percentage of the memory will be used as the cache. The default value is 30%. So make sure it will not affect the role business execution. The dedicate mode will use all memory in the virtual machine as the cache. In fact it will reserve some for operation system, azure hosting etc.. But it will try to use as much as the available memory to be the cache. As you can see, the Caching (Preview) was defined based on roles, which means all instances of this role will apply the same setting and play as a whole cache pool, and you can consume it by specifying the name of the role, which I will demonstrate later. And in a windows azure project we can have more than one role have the Caching (Preview) enabled. Then we will have more caches. For example, let’s say I have a web role and worker role. The web role I specified 30% co-located caching and the worker role I specified dedicated caching. If I have 3 instances of my web role and 2 instances of my worker role, then I will have two caches. As the figure above, cache 1 was contributed by three web role instances while cache 2 was contributed by 2 worker role instances. Then we can add items into cache 1 and retrieve it from web role code and worker role code. But the items stored in cache 1 cannot be retrieved from cache 2 since they are isolated. Back to our Visual Studio we specify 30% of co-located cache and use the local storage emulator to store the cache cluster runtime status. Then at the bottom we can specify the named caches. Now we just use the default one. Now we had enabled the Caching (Preview) in our web role settings. Next, let’s have a look on how to consume our cache.   Consume Caching (Preview) The Caching (Preview) can only be consumed by the roles in the same cloud services. As I mentioned earlier, a cache contributed by web role can be connected from a worker role if they are in the same cloud service. But you cannot consume a Caching (Preview) from other cloud services. This is different from the Shared Caching. The Shared Caching is opened to all services if it has the connection URL and authentication token. To consume the Caching (Preview) we need to add some references into our project as well as some configuration in the Web.config. NuGet makes our life easy. Right click on our web role project and select “Manage NuGet packages”, and then search the package named “WindowsAzure.Caching”. In the package list install the “Windows Azure Caching Preview”. It will download all necessary references from the NuGet repository and update our Web.config as well. Open the Web.config of our web role and find the “dataCacheClients” node. Under this node we can specify the cache clients we are going to use. For each cache client it will use the role name to identity and find the cache. Since we only have this web role with the Caching (Preview) enabled so I pasted the current role name in the configuration. Then, in the default page I will add some code to show how to use the cache. I will have a textbox on the page where user can input his or her name, then press a button to generate the email address for him/her. And in backend code I will check if this name had been added in cache. If yes I will return the email back immediately. Otherwise, I will sleep the tread for 2 seconds to simulate the latency, then add it into cache and return back to the page. 1: protected void btnGenerate_Click(object sender, EventArgs e) 2: { 3: // check if name is specified 4: var name = txtName.Text; 5: if (string.IsNullOrWhiteSpace(name)) 6: { 7: lblResult.Text = "Error. Please specify name."; 8: return; 9: } 10:  11: bool cached; 12: var sw = new Stopwatch(); 13: sw.Start(); 14:  15: // create the cache factory and cache 16: var factory = new DataCacheFactory(); 17: var cache = factory.GetDefaultCache(); 18:  19: // check if the name specified is in cache 20: var email = cache.Get(name) as string; 21: if (email != null) 22: { 23: cached = true; 24: sw.Stop(); 25: } 26: else 27: { 28: cached = false; 29: // simulate the letancy 30: Thread.Sleep(2000); 31: email = string.Format("{0}@igt.com", name); 32: // add to cache 33: cache.Add(name, email); 34: } 35:  36: sw.Stop(); 37: lblResult.Text = string.Format( 38: "Cached = {0}. Duration: {1}s. {2} => {3}", 39: cached, sw.Elapsed.TotalSeconds.ToString("0.00"), name, email); 40: } The Caching (Preview) can be used on the local emulator so we just F5. The first time I entered my name it will take about 2 seconds to get the email back to me since it was not in the cache. But if we re-enter my name it will be back at once from the cache. Since the Caching (Preview) is distributed across all instances of the role, so we can scaling-out it by scaling-out our web role. Just use 2 instances and tweak some code to show the current instance ID in the page, and have another try. Then we can see the cache can be retrieved even though it was added by another instance.   Consume Caching (Preview) Across Roles As I mentioned, the Caching (Preview) can be consumed by all other roles within the same cloud service. For example, let’s add another web role in our cloud solution and add the same code in its default page. In the Web.config we add the cache client to one enabled in the last role, by specifying its role name here. Then we start the solution locally and go to web role 1, specify the name and let it generate the email to us. Since there’s no cache for this name so it will take about 2 seconds but will save the email into cache. And then we go to web role 2 and specify the same name. Then you can see it retrieve the email saved by the web role 1 and returned back very quickly. Finally then we can upload our application to Windows Azure and test again. Make sure you had changed the cache cluster status storage account to the real azure account.   More Awesome Features As a in-memory distributed caching solution, the Caching (Preview) has some fancy features I would like to highlight here. The first one is the high availability support. This is the first time I have heard that a distributed cache support high availability. In the distributed cache world if a cache cluster was failed, the data it stored will be lost. This behavior was introduced by Memcached and is followed by almost all distributed cache productions. But Caching (Preview) provides high availability, which means you can specify if the named cache will be backup automatically. If yes then the data belongs to this named cache will be replicated on another role instance of this role. Then if one of the instance was failed the data can be retrieved from its backup instance. To enable the backup just open the Caching page in Visual Studio. In the named cache you want to enable backup, change the Backup Copies value from 0 to 1. The value of Backup Copies only for 0 and 1. “0” means no backup and no high availability while “1” means enabled high availability with backup the data into another instance. But by using the high availability feature there are something we need to make sure. Firstly the high availability does NOT means the data in cache will never be lost for any kind of failure. For example, if we have a role with cache enabled that has 10 instances, and 9 of them was failed, then most of the cached data will be lost since the primary and backup instance may failed together. But normally is will not be happened since MS guarantees that it will use the instance in the different fault domain for backup cache. Another one is that, enabling the backup means you store two copies of your data. For example if you think 100MB memory is OK for cache, but you need at least 200MB if you enabled backup. Besides the high availability, the Caching (Preview) support more features introduced in Windows Server AppFabric Caching than the Windows Azure Shared Caching. It supports local cache with notification. It also support absolute and slide window expiration types as well. And the Caching (Preview) also support the Memcached protocol as well. This means if you have an application based on Memcached, you can use Caching (Preview) without any code changes. What you need to do is to change the configuration of how you connect to the cache. Similar as the Windows Azure Shared Caching, MS also offers the out-of-box ASP.NET session provider and output cache provide on top of the Caching (Preview).   Summary Caching is very important component when we building a cloud-based application. In the June 2012 update MS provides a new cache solution named Caching (Preview). Different from the existing Windows Azure Shared Caching, Caching (Preview) runs the cache cluster within the role instances we have deployed to the cloud. It gives more control, more performance and more cost-effect. So now we have two caching solutions in Windows Azure, the Shared Caching and Caching (Preview). If you need a central cache service which can be used by many cloud services and web sites, then you have to use the Shared Caching. But if you only need a fast, near distributed cache, then you’d better use Caching (Preview).   Hope this helps, Shaun All documents and related graphics, codes are provided "AS IS" without warranty of any kind. Copyright © Shaun Ziyan Xu. This work is licensed under the Creative Commons License.

    Read the article

  • C# CreatePipe() -> Protected memory error

    - by M. Dimitri
    Hi all, I trying to create a pipe using C#. The code is quite simple but I get a error saying "Attempted to read or write protected memory. This is often an indication that other memory is corrupt." Here the COMPLETE code of my form : public partial class Form1 : Form { [DllImport("kernel32.dll", CharSet = CharSet.Auto, SetLastError = true)] public static extern bool CreatePipe(out SafeFileHandle hReadPipe, out SafeFileHandle hWritePipe, SECURITY_ATTRIBUTES lpPipeAttributes, int nSize); [StructLayout(LayoutKind.Sequential)] public struct SECURITY_ATTRIBUTES { public DWORD nLength; public IntPtr lpSecurityDescriptor; public bool bInheritHandle; } public Form1() { InitializeComponent(); } private void btCreate_Click(object sender, EventArgs e) { SECURITY_ATTRIBUTES sa = new SECURITY_ATTRIBUTES(); sa.nLength = (DWORD)System.Runtime.InteropServices.Marshal.SizeOf(sa); sa.lpSecurityDescriptor = IntPtr.Zero; sa.bInheritHandle = true; SafeFileHandle hWrite = null; SafeFileHandle hRead = null; if (CreatePipe(out hRead, out hWrite, sa, 4096)) { MessageBox.Show("Pipe created !"); } else MessageBox.Show("Error : Pipe not created !"); } } At the top I declare : using DWORD = System.UInt32; Thank you very much if someone can help.

    Read the article

< Previous Page | 34 35 36 37 38 39 40 41 42 43 44 45  | Next Page >